Сохранен 533
https://2ch.hk/spc/res/176350.html
Домены arhivach.top и arhivach.site временно не функционируют! Используйте домен ARHIVACH.XYZ.
24 декабря Архивач восстановлен после серьёзной аварии. К сожалению, значительная часть сохранённых изображений и видео была потеряна. Подробности случившегося. Мы призываем всех неравнодушных помочь нам с восстановлением утраченного контента!

Тред тупых вопросов и умных ответов. #25

 Аноним 06/05/15 Срд 13:24:15 #1 №176350 
14309078550320.jpg
Тред вопросов о жизни, Вселенной и всем таком.

Спрашиваем то, за что в других местах выдают путёвку в биореактор. Здесь анонимные ученые мирового уровня критически рассмотрят любые гениальные идеи и нарисованные в Paint схемы.

Прошлый тред тут: https://2ch.hk/spc/res/167732.html
Аноним 06/05/15 Срд 13:26:47 #2 №176353 
Что будет после тепловой смерти вселенной?
Аноним 06/05/15 Срд 13:28:38 #3 №176354 
>>176353
Сингулярность.
Аноним 06/05/15 Срд 13:29:17 #4 №176356 
>>176354
А потом
Аноним 06/05/15 Срд 13:35:01 #5 №176361 
>>176353
Тепловой смерти не будет. Эта гипотеза базируется на устаревших знаниях и не учитывает множество факторов.
Аноним 06/05/15 Срд 14:02:29 #6 №176375 
>>176356
Это не наука уже, а пустая болтовня - философия. Вот ответ на твой вопрос - бесконечно малые величины, это тогда, когда производный результат рассматривается как бесконечная сумма бесконечно малых. Тоже самое относится к бесконечно большим величинам. Это не сингулярность ибо, на определённом локальном отрезке числовой прямой можно посчитать координаты. Вселенная локальна как процесс между + бесконечность, - бесконечность. И движется как процесс в ту или иную сторону.
Аноним 06/05/15 Срд 14:05:24 #7 №176377 
Можно ли уничтожить чёрную дыру?
Аноним 06/05/15 Срд 14:10:09 #8 №176382 
>>176377
Создать вокруг неё непроницаемый экран и ждать пока она испарится
Аноним 06/05/15 Срд 14:16:51 #9 №176386 
>>176377
Чёрная дыра это - процесс, описываемый математической моделью. Черная дыра испаряется медленно, но верно. Чёрная дыра как термин не имеет смысла. Радиус Шварцшильда имеет смысл, так как, математически моделируется.
Аноним 06/05/15 Срд 17:15:03 #10 №176452 
>>176386
Два чая этому астрофизику.
Аноним 06/05/15 Срд 18:29:07 #11 №176472 
>>176377
Существует ли в природе антимасса? Что будет если в чёрную дыру скинуть равную массе дыры антимассу?
Аноним 06/05/15 Срд 18:37:55 #12 №176475 
>>176472
>Что будет если в чёрную дыру скинуть равную массе дыры антимассу?
Ниче не будет, она не станет падать в дыру - ее суть отталкиваться от нормальной массы.
Аноним 06/05/15 Срд 20:40:16 #13 №176565 
>>176475
Если сделать костюм из антимассы, можно будет летать?
Аноним 06/05/15 Срд 21:07:15 #14 №176577 
>>176565
Нет.
Аноним 06/05/15 Срд 21:08:11 #15 №176579 
>>176577
Почему
Аноним 06/05/15 Срд 21:14:01 #16 №176580 
>>176350
Подскажите годных регулярнообновляющихся ресурсов про космос - основное направление экхопланеты, но это не принципиально. Можно и на английском.
Из того, что у меня есть
НЕ КИДАЙТЕ ГОВНОМ ОЧЕНЬ СИЛЬНО

http://www.astronews.ru/
http://planetquest.jpl.nasa.gov/
http://www.eso.org/public/
http://kepler.nasa.gov/
http://www.astrogorizont.com/
http://sdnnet.ru/
Аноним 06/05/15 Срд 21:57:09 #17 №176596 
>>176580
>основное направление экхопланеты
http://arxiv.org/list/astro-ph/recent
http://iopscience.org/aj
http://iopscience.org/apj
и т.п., также по касательной
http://www.sciencedirect.com/science/journal/00191035
http://www.sciencedirect.com/science/journal/00320633
Если надо более адаптировано:
http://exoplanet.hanno-rein.de/ каталог экзопланет, один из многих
http://phys.org/space-news/astronomy/
http://www.sciencedaily.com/news/space_time/extrasolar_planets/ (Science Daily вообще крутой)

> но это не принципиально
http://novosti-kosmonavtiki.ru/mag/ (охуенный журнал, много эксклюзивных подробностей)
http://www.nasaspaceflight.com/l2/ (аналогично)
http://www.thespacereview.com/ прекрасная аналитика
http://www.americaspace.com/
http://kosmolenta.com/
Аноним 06/05/15 Срд 22:05:56 #18 №176599 
>>176377
Я знаю ответ на этот вопрос, и таки отвечу, с точки зрения современной науки.
Внимание, правильный ответ:

Неизвестно.
Аноним 06/05/15 Срд 22:09:07 #19 №176601 
>>176579
Можно будет только улететь, но не летать. Это конечно оче улётно, но в центре эриданского супервойда совершенно нечего делать.
Аноним 06/05/15 Срд 22:11:15 #20 №176602 
>>176580
> экхопланеты
http://www.allplanets.ru/
Аноним 06/05/15 Срд 22:40:34 #21 №176617 
>>176475
Вещество с отрицательной массой будет притягиваться тоже, в этом-то и вся сложность её задетектить. Не слушайте дураков.
Аноним 07/05/15 Чтв 02:17:13 #22 №176678 
>>176617
В чем суть тогда?
Аноним 07/05/15 Чтв 04:00:20 #23 №176688 
Ксанфомалити - поехавший фрик от науки, или годный учёный?
Аноним 07/05/15 Чтв 04:16:57 #24 №176690 
>>176688
Он годный ученый, но если ты про форс жизни на Венере, то тут у Леонида Васильевича немного поехала кукушечка. Такое случается, он человек пожилой, 83 года дедушке. Может себе позволить и покуралесить на старости лет.
Аноним 07/05/15 Чтв 04:20:17 #25 №176691 
>>176690
А Морган Фримен?
Аноним 07/05/15 Чтв 04:20:51 #26 №176692 
>>176691
Морган Фримен - актер и телеведущий. Он шоумен, а не ученый.
Аноним 07/05/15 Чтв 04:36:00 #27 №176695 
>>176692
А че он про космос тогда рассказывает? Может немного учёный?
Аноним 07/05/15 Чтв 05:19:44 #28 №176698 
>>176695
Он ведет развлекательную педерачу, с научно-популярным уклоном, в которой пугает быдло черными дырами и нейтронными звездами. Сценарий и текст он не пишет, за него это делают специально обученные люди. Он продюсер и ведущий, в его обязанности входит проникновенно доносить до ЦА всю хуйню, что ему написали, разбавляя эти телеги охуительными историями про свое детство.
Аноним 07/05/15 Чтв 19:33:16 #29 №176910 
Как космонавты справляются с долгим воздержанием? Таблетки какие пьют или фапают?
Слышал на борту шаттла оргии устраивали - правда?
Аноним 07/05/15 Чтв 21:55:44 #30 №177022 
14310249441050.jpg
У чёрной дыры есть горизонт событий - гипотетическая граница из-за которой уже ничего не может вырваться. Если верить физикам-теоретикам, то при пересечении этой линии любое тело начинает очень сильно растягиваться. Тогда вопрос: если я стою рядом с горизонтом и пропускаю за него шестую часть деревянной палки, которую затем тут же тащу назад, что произойдёт с палкой?
Аноним 07/05/15 Чтв 21:59:01 #31 №177024 
>>177022
Ты ее не вытащишь
Аноним 07/05/15 Чтв 22:05:43 #32 №177025 
>>177022
Палка тебя утащит за собой.
Аноним 07/05/15 Чтв 22:13:23 #33 №177027 
>>177022
Допустим, ты нашел совсем не вращающуюся ЧД. Тогда твоя картинка не релейтед.
Но это все равно, с палкой произойдет спагеттизация, материал совершенно не важен, хоть деревянная, хоть стеклянная, хоть оловянная. Хоть адамантовая, хоть криптонитовая, хоть их аллаха. Разница только в том, что на вращающуюся дыру палку будет наматываться спиралью, даже если ты сумеешь остаться неподвижным, а в невращающуюся дыру палка будет вытягиваться ровно.
Аноним 07/05/15 Чтв 22:17:33 #34 №177030 
Допустим есть идеально твердая твердая неразрушимая палка длиной в несколько световых лет. Если подвергать,на том конце примут сигнал почти сразу? ТО разрушена?
Аноним 07/05/15 Чтв 23:51:30 #35 №177069 
14310318906860.jpg
>>177030
>Допустим есть идеально твердая твердая неразрушимая палка длиной в несколько световых лет. Если подвергать,на том конце примут сигнал почти сразу? ТО разрушена?
Платиновые вопросы за 500.
Нет никаких неразрушаемых палок, не существует. Скорость взаимодействия между частицами, из которых состоит палка, передается с конечной скоростью, так что наебать ТО не получится.
Аноним 08/05/15 Птн 00:01:05 #36 №177071 
>>176580
http://astrobiology.com/
http://spaceref.com/
http://nasawatch.com/
Аноним 08/05/15 Птн 12:32:11 #37 №177244 
>>176617
Что такое отрицательная масса, и чем она отличается от положительной?
Аноним 08/05/15 Птн 12:33:46 #38 №177246 
>>177244
Примерно как жидкий вакуум отличается от твердого.
Аноним 08/05/15 Птн 12:34:43 #39 №177248 
>>177246
Это слишком умный ответ, ты ошибся тредом.
Аноним 08/05/15 Птн 15:50:30 #40 №177291 
14310894307790.jpg
Это охуенно. Я конкретно понял процесс формирования планет из планетезималей и адгезии с точки зрения именно механики. Вообще охуенный курс, советую новичкам присоединиться, лекчурер тоже харизматичный, не Фейнман, конечно, но всё же.
https://class.coursera.org/solarsystem-002/lecture/99
Аноним 08/05/15 Птн 16:54:57 #41 №177308 
>>177291
они хотят чтобы я залогинился, я хз
если кто залогинится, то вы по хардкору годно там или нет, а то анон чтото заинтриговал
Аноним 08/05/15 Птн 18:07:57 #42 №177329 
14310976771990.jpg
>>177308
Там не только залогиниться надо, но и подписаться на курс. https://www.coursera.org/course/solarsystem -- здесь подписаться кнопкой "join course", потом уже можно открывать данную аноном ссылку на конкретную лекцию (иначе сайт перенаправляет на свою главную страницу).

>>177291
Так это же Майк "убийца Плутона" Браун, чья команда открыла кучу транснептуновых объектов! Шикарная лекция, анон, буду слушать остальное. Добра тебе.
Аноним 09/05/15 Суб 18:51:56 #43 №177726 
14311867164440.gif
Устойчивое или не устойчивое равновесие тел в точках Лагранжа? Каким образом стабилизируют положение космического аппарата в точках Лагранжа? Почему Троянцы никуда не разлетаются из точки Лагранжа? Троянцы вращаются вокруг своего общего центра масс? И как они так четко попали в точку Лагранжа во время образования образования вселенной?
Аноним 09/05/15 Суб 18:56:22 #44 №177727 
>>177726
1 Неустойчивое.
2 Круги наматывают вокруг точки
3 Вращаются вокруг точки.
4 Захват, или сгребание попутного мусора
Аноним 09/05/15 Суб 19:13:12 #45 №177729 
>>177726
Они кружатся не вокруг точки, а вокруг общего центра масс. Т.е. вокруг солнца. Незначительное притяжение тела Б , изменяет орбитальную скорость пренебрежительно малого объекта, так что оно попадает в орбитальный резонанс 1:1.
Аноним 09/05/15 Суб 20:45:06 #46 №177744 
>>177727
Не пизди. В L4 и L5 устойчивое.
По остальным пунктам ты соответственно обосрался.
Аноним 09/05/15 Суб 21:29:06 #47 №177760 
>>177726
>Каким образом стабилизируют положение космического аппарата в точках Лагранжа?
В случае L1, например, оно обращается по псевдостабильной гало-орбите с радиусом в несколько десятков тысяч км (обычно). Псевдостабильная потому, что с неё аппарат сбивают возмущения. На таких удалениях от L1 на возвращение к идеальной траектории тратится совсем немного дельты, собственно мелкими коррекциями аппарат и удерживается. Все это происходит под контролем баллистиков, обеспечивающих навигацию. В GMAT есть семпл того, как примерно выглядит такая траектория при активном маневрировании, вот видео из него. (каждый виток занимает около 4 месяцев в данном случае, на его протяжении делается несколько коррекций)
Аноним 09/05/15 Суб 22:36:08 #48 №177786 
>>177744
Обосрался - это не аргумент, если у тебя есть другой, правильный ответ, то потрудись написать.
Аноним 10/05/15 Вск 03:09:38 #49 №177859 
Я слышал были успешные запуски солнечных парусов. Насколько годно получилось, какая тяга, перспективы?
Аноним 10/05/15 Вск 03:15:49 #50 №177861 
>>177859
http://www.jspec.jaxa.jp/e/activity/ikaros.html
Аноним 10/05/15 Вск 14:33:44 #51 №177967 
Если астронавт упадёт с орбиты, то он сгорит в атмосфере?
Аноним 10/05/15 Вск 14:36:05 #52 №177969 
>>177967
>упадёт с орбиты
А если не упадёт?
Аноним 10/05/15 Вск 14:36:33 #53 №177971 
>>177967
Разумеется да.
Аноним 10/05/15 Вск 14:37:46 #54 №177972 
>>177969
В гравитации упал же.
>>177971
Но почему? Из стратосферы прыгали с парашютом и всё нормально.
Аноним 10/05/15 Вск 14:43:49 #55 №177975 
>>177972
>В гравитации упал же.
Кто?
Труп Клуни остался на орбите, тёлка была в капсуле.

Да и не стоит брать за пример кино где мусор летает по одной и той же орбите со спутниками с высокой относительной скоростью.
Аноним 10/05/15 Вск 14:50:45 #56 №177977 
>>177975
Клуни притянула гравитация, он же специально от ремней отцепился, чтобы её за собой не утянуть.
Так почему астронавт сгорит?
>где мусор летает по одной и той же орбите со спутниками с высокой относительной скоростью
Как вообще запускают спутники, если на орбите куча летающего мусора и метеоритов?
Аноним 10/05/15 Вск 15:13:00 #57 №177988 
>>177977
> Клуни притянула гравитация
Вот по этому и не стоит смотреть подобные фильмы.

Аноним 10/05/15 Вск 15:31:58 #58 №177992 
>>177988
Ну а с астронавтом что будет? Любой предмет что ли независимо от размера и массы сгорит в атмосфере падая на Землю из космоса?
Аноним 10/05/15 Вск 15:35:18 #59 №177994 
>>177992
Метеоры - сгорают.
Метеориты - таки достигают земли.

Ну а что уж говорить о простых людишках из воды.

https://ru.wikipedia.org/wiki/%D0%9C%D0%B5%D1%82%D0%B5%D0%BE%D1%80%D0%B8%D1%82#.D0.9F.D1.80.D0.BE.D1.86.D0.B5.D1.81.D1.81_.D0.BF.D0.B0.D0.B4.D0.B5.D0.BD.D0.B8.D1.8F_.D0.BC.D0.B5.D1.82.D0.B5.D0.BE.D1.80.D0.BD.D1.8B.D1.85_.D1.82.D0.B5.D0.BB_.D0.BD.D0.B0_.D0.97.D0.B5.D0.BC.D0.BB.D1.8E
Аноним 10/05/15 Вск 15:37:25 #60 №177995 
14312614455590.png
>>177977
Если падать с орбиты, то сгоришь если со стратостата нет. Орбитальная скорость выше 8 км\с . Что бы вернуться на землю, надо её сбросить. Допустим ты сбросил её(скорость) и теперь летишь по баллистической траектории. Твоя скорость всё такая же большая + ты летишь под углом. И если у тебя нет дециметровой обшивки из тефлона, ты сгоришь из за трения. Если же падать отвесно то, либо не успеешь затормозить об атмосферу и парашют порвётся, либо ты так же сгоришь на меньших скоростях.
Аноним 10/05/15 Вск 16:32:35 #61 №178008 
14312647550170.gif
>>177972 >>177967 >>177992
>Из стратосферы прыгали с парашютом и всё нормально.
Стратосфера - это 40км вверх. Орбита - это 7.9км/сек вбок. Чуешь разницу? Чувак, падающий с 40км, едва 350м/сек развивает.

Чисто теоретически можно захуячить "космический парашют" в виде зонтика залупой вперед, сильно увеличивающий драг и ослабляющий нагрев до уровня, который держат даже полимеры вроде кевлара (<400 C). Но точность посадки будет хуевая, и окно входа в атмосферу оче узкое, и вообще там масса ограничений. Такие испытывались с переменным успехом - НТУ для возвращения Фрегата, IRDT, HIAD. Всерьез для Земли такое не применяется пока.
Аноним 10/05/15 Вск 18:33:06 #62 №178046 
>>177992
>Любой предмет что ли независимо от размера и массы сгорит в атмосфере падая на Землю из космоса?
Нет, маленькие не згорят. Вот например иголки-антенны выпали целенькие >>151982 очевидно потому что легко затормозились в атмосфере благодаря малой массе. Так же выпадает всякая космическая пыль
>Косми́ческая пыль образуется в космосе частицами размером от нескольких молекул до 0,2 мкм. 40 000 тонн космической пыли каждый год оседает на планете Земля
https://ru.wikipedia.org/wiki/Космическая_пыль
Если зделать достаточно тяжелое тело, но с большой поверхностью то очевидно тоже не згорит, развернутая тряпка например, пленка или фольга, фанера какая-нить, оно само себе парашут. Однако энтузиасты думают как прыгать с космоса в спец скафандрах http://www.membrana.ru/particle/3199
Аноним 10/05/15 Вск 18:34:33 #63 №178048 
>>177995
>Если же падать отвесно то, либо не успеешь затормозить об атмосферу и парашют порвётся, либо ты так же сгоришь на меньших скоростях.
То есть даже если падать как обычный парашютист вертикально вниз, всё равно сгоришь?
Аноним 10/05/15 Вск 18:50:44 #64 №178051 
>>178008
Ну а если просто сбросить скорость?
Аноним 10/05/15 Вск 18:54:38 #65 №178052 
>>178048
Вертикально вниз - это не сход с орбиты уже,
это просто падение. Чтобы так "сойти с орбиты", надо каким-то макаром погасить 7.9км/с орбитальной скорости.

Скорость входа в плотные слои тут будет зависеть больше от высоты, с которой падаешь. Я не берусь оценить высоту, с которой можно прыгнуть не сгорев и не расплющившись, это нужна симуляция, не очень тривиальная.
Аноним 10/05/15 Вск 22:29:58 #66 №178124 DELETED
>>178051
Просто это как? Ручник дернуть?
Аноним 10/05/15 Вск 22:34:52 #67 №178127 
>>178124>>178052>>178046
Можно ведь ещё в космосе открыть парашют и плавно входя в плотные слои постепенно затормозиться с постепенно нарастающим давлением на парашют.
Аноним 10/05/15 Вск 22:52:47 #68 №178135 
Что там с излучением, из-за которого не получится полететь на марс? Фэйк или нет?
Аноним 10/05/15 Вск 22:54:58 #69 №178138 
>>178135
Зачем на него лететь? Там же гравитация слабее даже.
Аноним 10/05/15 Вск 23:00:32 #70 №178142 DELETED
>>178127
Ты хоть понимаешь что несешь? Парашют не будет работать пока не окажется в плотных слоях атмосферы
Аноним 10/05/15 Вск 23:03:19 #71 №178144 
>>178142
Как будто его нельзя раскрыть капсулами со сжатым газом.
Аноним 10/05/15 Вск 23:03:58 #72 №178145 
>>178138
Ну как зачем? Мечты о космических просторах, планетах, вот. НАСА запилила грант же на решение проблемы радиации при полетах. Тут даже говорят, что высадка на луне - фэйк, в частности из-за излучения.
Аноним 10/05/15 Вск 23:09:15 #73 №178154 DELETED
>>178144
Можно. Но работать он все равно не будет
Аноним 10/05/15 Вск 23:12:08 #74 №178155 
>>178154
Он будет работать постепенно по мере вхождения в плотные слои. торможение будет плавным и не порвёт парашют.
Аноним 10/05/15 Вск 23:21:50 #75 №178160 
>>178145
Ты согласишься жить на Марсе? Гравитация в 3 раза слабее нашей, люди деградируют в физическом плане. И ещё дикий холод.
Аноним 10/05/15 Вск 23:22:40 #76 №178163 DELETED
>>178155
Какие же тупые инженеры в космической промышленности
За 60 лет нишмагли придумать такой йоба-парашют
Может ЗАГОВОР?
Аноним 10/05/15 Вск 23:41:33 #77 №178173 
>>178144
>Как будто его нельзя раскрыть капсулами со сжатым газом.
А кстати. Что если надуть большую сферу, или другой формы фигуру? По мере вхождения в плотные слои поддувать из баллона газ чтобы уравнивать с давлением среды. Она же будет тормозить хорошо уже на низкой орбите, а потом довольно плавно упадет вниз. Но это видимо немаленькая и нелегкая конструкция для современных запусков.
Аноним 10/05/15 Вск 23:45:39 #78 №178174 
>>178160
>Гравитация в 3 раза слабее нашей
Наша гравитация это причина множества заболеваний сердечно-сосудистой системы и опорно-двигательного аппарата. Заметь - никакие звери прямохождение не закрепили в процессе эволюции, кроме части высших обезьян.
Аноним 10/05/15 Вск 23:49:39 #79 №178177 DELETED
>>178173
Есть некоторая зависимость между орбитальной скоростью, ускорением свободного падения, и плотностью атмосферы. И между первыми двумя и атмосферой нихуя не линейная зависимость.
Аноним 10/05/15 Вск 23:49:51 #80 №178178 
>>178163
А кому это надо? Никто же не будет бросать шаттл в космосе.
Аноним 10/05/15 Вск 23:56:36 #81 №178182 DELETED
>>178178
Не вижу связи. Да загугли наконец что такое орбитальный полёт.
Аноним 11/05/15 Пнд 00:10:17 #82 №178190 
>>178182
Зачем астронавтам десантироваться, если они возвращаются на землю на корабле?
Аноним 11/05/15 Пнд 00:11:08 #83 №178191 
В воркач неохота идти, по-моему тут более прошаренный анон в этой теме сидит, так что у меня такой вопрос. Реально ли устроиться в корпорации типа Lockheed Martin в какой-нибудь отдел клепания космических йоб, имея простой рашкинский диплом? Или туда берут только элитку из универов типа MIT и т.д.
Аноним 11/05/15 Пнд 00:12:26 #84 №178193 
>>178190
Что бы корабль не сбили.
Аноним 11/05/15 Пнд 00:17:37 #85 №178196 
>>178193
А затормозить как без корабля?
Аноним 11/05/15 Пнд 00:19:52 #86 №178198 
>>178191
Помню лет 10-15 назад по телеку Велихов (он сейчас рулит Итером) перетирая за образование говорил что в Америке люди делают спутники без ВО в нашем понимании.
Аноним 11/05/15 Пнд 00:21:32 #87 №178199 
>>178198
>в Америке люди делают спутники без ВО в нашем понимании
Битарды форчана?
Аноним 11/05/15 Пнд 00:22:09 #88 №178200 
>>178196
Прыгать с эфирным парашютом.
Аноним 11/05/15 Пнд 00:23:15 #89 №178201 
>>178177
Если прогресс с дидовским знаменем раздулся бы до 50 метров в диаметре что бы было?
Аноним 11/05/15 Пнд 00:24:04 #90 №178203 DELETED
>>178196
Ручник видимо дернуть. Тот анон нихуя не понимает что орбитальное движение суть падение с первой космической скоростью. И если затормозиться то гравитация пересилит и земляшка начнёт притягивать обьект, чем меньше скорость тем быстрее падение на землю.
Аноним 11/05/15 Пнд 00:27:51 #91 №178208 
14312932716460.jpg
>>178174
>Заметь - никакие звери прямохождение не закрепили в процессе эволюции, кроме части высших обезьян.
Тебя сейчас этот парень ударит. Алсо, к ним уже коты подтягиваются.

>>178196
Обожраться гороха и пердеть в сторону земляшки.
Аноним 11/05/15 Пнд 00:56:11 #92 №178216 
14312949715170.jpg
14312949715201.jpg
>>178208
У твоего оленя просто передние конечности рахитичные.
Аноним 11/05/15 Пнд 00:57:34 #93 №178217 
>>178208
Они не прямоходящие. Они бипедальные, как динозавры и страусы, но это не прямохождение, так как их тело висит над ногами буквой Т.
Только обезьяны развили именно прямохождение, когда тело стоит на ногах буквой I. Это нетривиальная разница, потому что такое положение тела без дополнительных эволюционных перестроек, которые претерпел человек, очень неустойчиво.
Аноним 11/05/15 Пнд 01:07:04 #94 №178219 
>>178127
>плавно входя в плотные слои постепенно
вот с этой частью и проблема, плотность атмосферы падает по экспоненте с увеличением высоты, и соотвественно при снижении увеличивается не линейно. драг правда пропорционален квадрату скорости, те падает тоже относительно быстро.
вообщем вопрос нахождения баланса. если у тебя есть еба которая из носового платка вырастает в квадратный километр, держит 1000-2000 градусов, достаточно прочна - то да де́сант приземлится
Аноним 11/05/15 Пнд 01:07:31 #95 №178220 
14312956515310.jpg
>>178173
>>178142
Это правильная идея. Гиперзвуковых парашютов (точнее "надувных замедлителей") бывает 2 типа, их еще с 60-х испытывают и юзают для входа АМС в атмосферы планет - с наддувом баллончиком, и с наддувом набегающим потоком. У каждого свои плюсы и минусы. Если вопрос интересен, рекомендую прочесть например пиздатейшую книжку Planetary Landers and Entry Probes, где этот вопрос разбирается более детально.
http://www.amazon.com/Planetary-Landers-Entry-Probes-Andrew/dp/0521820022
И вообще, книжка много аспектов раскрывает без какого-то супер-йобистого матана, будет полезно всем спейсачерам.

>>178201
Он бы не сгорел, скорее всего. 50м в диаметре при той же массе это дохуя.

>>178051
Проблема в том, что этой скорости 7900м/с, это огромная цифра, которую просто неоткуда взять в реактивном аппарате. Если у тебя есть куча лишней характеристической скорости - можно просто всю дорогу снижаться, помогая движком. Или вообще на орбиту не выходить, просто вися в вакууме с читом на бесконечное топливо, лол.

Справедливости ради, некоторые таки пользуются похожим подходом, например первая ступень SpaceX снижает нагрев при входе в атмосферу, подтормаживая двигателем. Но там и ступень пустая к тому времени, и скорости намного меньше орбитальных.
http://www.youtube.com/watch?v=riU3DZmU-jE
Аноним 11/05/15 Пнд 01:13:38 #96 №178223 
>>178219
>держит 1000-2000 градусов
Меньше же, площадь увеличивается же.
Аноним 11/05/15 Пнд 01:18:16 #97 №178227 
>>178219
>вот с этой частью и проблема, плотность атмосферы падает по экспоненте с увеличением высоты, и соотвественно при снижении увеличивается не линейно.
Вот поэтому юзают не баллистический спуск, а управляемый планирующий. Любой кирпич будет летать, если его разогнать достаточно быстро. Проблема в том, что на гиперзвуке почти всё летает как кирпич, т.е. имеет хуевый аэродинамический коэффициент. Так что правильная капсула может быть даже лучше самолета в этом плане.

С планирующим спуском проблема в том, что точность приземления низкая, и окно входа в атмосферу очень узкое, надо выдрачивать траекторию для минимизации нагрева и нагрузок. Но сделав всё правильно, можно даже без теплозащиты как таковой обойтись. Что было экспериментально подтверждено как минимум один раз при спуске НТУ, например. (который проебался при посадке, но после активной фазы реентри таки успел передать телеметрию).
Аноним 11/05/15 Пнд 01:24:57 #98 №178229 
>>178191
Реально, если ты сейчас школьник, и будешь целенаправленно въебывать 24/7 для этого.

>>178173
Альтернативных концептов без-нагревного входа в атмосферу немало. Тросовый замедлитель, например (за капсулой болтается 20км трос, тупо за счет длины создает трение), решетчатые крылья (которые хорошо себя ведут на высоких скоростях и за это давно применяются в ракетах) и т.п. Ни одна из них, кроме надувных замедлителей, пока не была воплощена в реале.
Аноним 11/05/15 Пнд 01:26:15 #99 №178230 
>>178227
>аэродинамическое качество
очевидный фикс
Аноним 11/05/15 Пнд 01:39:27 #100 №178234 
14312975671110.jpg
>>177859
Но я не знаю ангельский
Аноним 11/05/15 Пнд 02:10:31 #101 №178240 
>>178229
С сожалению, уже давно не школьник, скоро заканчивать российский вуз буду с весьма посредственными оценками. Очень огорчаюсь от того, что не начал въёбывать 24\7 с школьных лет, да и тогда всё это казалось совсем нереальным, а сейчас поработав на дохуище рандомных работ окончательно осознал, что хочу пилить космические йобы
мне хотяб в сборочный цех, болты крутить, эх
Аноним 11/05/15 Пнд 14:14:07 #102 №178325 
>>178223
ну я тут, даже не лаптем, прикинул
наш де́сант с экзоскелетом и пушкой пол тонны пусть весит, реентири гдето 5 минут(многовато для тупого входа, но ок пусть будет), 2.5g перегрузки. на начальном этапе придется рассеивать 100 мегаджоулей в сек. проблема ебы, то что ее собственная теплоемкость в этих условиях стремиться к нулю(физику не наебешь теплоемкость идет на килограммы, пусть у нас 100кг этой еба материи), те мы не расчитываем на абеляционные свойства, а рассчитываем на унос этой энергии набегающим потоком.
на 250 км плотность воздуха просто никакущая (лаптем 10е-13 от нормы), лаптем: драг пропорционален произведению плотности, прощади и квадрату скорости. в этих условиях даже квадрат скорости не очень помогает.
ок тогда со 160 начнем, плюс минус порядок плотность 2х10е-9 от нормы. и тут я понимаю что хуй я так просто лаптем посчитаю это, поэтому буду считать левой куринной лапой (фантазировать как хочу, после прочтения Planetary_Landers_and_Entry_Probes.pdf ворвусь снова) - с одной стороны нам требуется сила 12500 ньютонов, дальше положа болт на аэтодинамику будем считать по энергиям.
очевидно чем выше температура, до которой нагреем набегающий воздух, тем больше энергии мы слили, 1 кг воздуха с дельтой температуры 1000 грудусов уносит 1 МДж (примерно), нам нужна сотка это 250х250 метров квадрат (тут летят тапки вес паруса получается 1.6Г на метр квадратный, нагрузка 0.2Н на метр , но мне пофиг я из сайфая, нанафибра все дела, хотя следует отметить что это раз в 5 меньше чем японский солнечный парус их икаруса, те десант не только может приземляться с такой ебой, но и летать по солнечной системе)
как только десант начал снимать орбитальную скорость, с этого момента он начинает приобретать скорость в направлении центра земли, и ускорении с которым он начинает это делать опять не линейно, те скинув скорость на 10 процентов(800 метров) он падает с ускорением 0.2g, 30 процентов и 0.5g, в два раза и он падает уже 0.75g. а пройденное расстояние при постоянном ускорении пропорционально квадрату времени, в нашем случае это расстояние есть высота, те высота уменьшается стремительно(время тикает, высота тикает в квадрате), и ужас плотность воздуха с уменьшением высоты растет еще стремительней (лаптем: каждые 8 км высоты растет в е(2.7) раз), помогает что скорость худо бедно но уменьшается и тот момент что сливать энергии нам надо меньше(по мере уменьшения скорости).
в любом случае наш де́сант не унывает, он же не тупой метеорит, и продолжает смолить свою трубку, четырехглазые все продумали, и по какой бы там формуле все не происходило, площадь аэродинамического сопротивления уменьшается за счет изменения парусности с пропорциональным увеличением прочности, сохраняя 12500 ньютонов драга.
хм ну тут еще надо накинуть массы для охлаждения самого экзоскелета, ну и тепловой щит какой никакой но иметь придеться, но я и так уже два часа пишу, такчто я пошел жрать, десант какраз к тому времени долетит - посмотрим
Аноним 11/05/15 Пнд 16:03:45 #103 №178364 
>>176350
Это же на оп-пике цитата из ресторана на краю вселенной? Не могу в инглиш.
Аноним 11/05/15 Пнд 16:27:00 #104 №178368 
>>178364
>оп-пике цитата из ресторана на краю вселенной?
Да.
Аноним 11/05/15 Пнд 18:20:39 #105 №178388 
>>176910
Работают. Серьёзно, если работа интересная, то можно и без бап прожить.
Аноним 11/05/15 Пнд 22:11:59 #106 №178427 
14313715197120.jpg
Нахуя нужна вот эта шторка на "Куполе"?
inb4 форточка, чтобы проветривать
Аноним 11/05/15 Пнд 22:27:44 #107 №178431 
>>178427
фотоаппарат крепить - не ?
Аноним 11/05/15 Пнд 23:43:26 #108 №178450 
>>178427
Что у них вместо стекла там? Насколько он прочное? Выдержит ли удар головой среднестатистического космонавта, запущенного в сторону иллюминатора?
Аноним 12/05/15 Втр 00:01:57 #109 №178458 
>>178450
Кварцевое стекло, толщиной 10 см.
Аноним 12/05/15 Втр 00:37:23 #110 №178478 
14313802432650.jpg
14313802432661.gif
14313802432662.jpg
>>178450
Стекло, только четырехслойное. Внутренний закаленный слой, два силовых слоя (один из них запасной), и внешний противоударный. На внутренний слой нанесено антибликовое покрытие, на внешний многослойное светофильтрующее (для защиты от ультрафиолета например), на промежуточные - покрытие против запотевания.
> Насколько он прочное?
Ему уже прилетело несколько MMODов, застряли в противоударном слое. В прошлом году было 2 удара, после которых решили держать противоударные створки закрытыми от греха подальше.
> Выдержит ли удар головой среднестатистического космонавта, запущенного в сторону иллюминатора?
Можно хоть башку расшибить, похуй.
Аноним 13/05/15 Срд 00:09:35 #111 №178754 
1. Возле каких типов звезд способны существовать жизнепригодные планеты?
2. Умрет ли Вселенная?
Аноним 13/05/15 Срд 00:27:56 #112 №178760 
>>178754
> 1. Возле каких типов звезд способны существовать жизнепригодные планеты?
Мы пока точно не знаем какие формы жизни возможны кроме белковой. Так что есть варианты. Основное условие - звезда должна быть достаточно старой и стабильной чтобы жизнь успела развиться
> 2. Умрет ли Вселенная?
Пока непонятно
Аноним 13/05/15 Срд 01:51:42 #113 №178778 
>>178754
2. Умрет, но неизвестно каким способом.
Аноним 13/05/15 Срд 02:08:39 #114 №178780 
14314721195800.png
>>178754
>1. Возле каких типов звезд способны существовать жизнепригодные планеты?
Если судить только по той жизни, что известна нам, то вот эти.
Аноним 13/05/15 Срд 08:26:43 #115 №178801 
Посоны, у меня 2 вопроса.

1) Смотрели фильм "гравитация"? Там был момент, когда тётка пытается подтянуть к себе мужика на длиннющих стропах от остатков мкс, но ей не хватает сил, и мужик в конце концов отпускает их и улетает. Вопрос: это нормально или чепуха? Ведь в космосе же невесомость, и по идее подтянуть к себе мужика должно было быть очень легко.

2) На орбите же есть гравитация? Небольшая. Я знаю про то, что чем дальше ты от земли, тем меньшее ускорение испытываешь и поэтому длинная палка в космосе будет медленно вращаться. Но есть же и какие-то другие силы? Силы, свяанные с движением по кругу. Центротремительное ускорение или типа того. Оно должно как то влиять на положение объектов внутри корабля, стоящего на орбите?
Аноним 13/05/15 Срд 10:11:55 #116 №178805 
>>178801
1) не смотрел и вряд ли буду, но если есть какое-то ускорение или вращение, то притянуть будет трудно. откуда, блядь, стропы на МКС

2) Приливные силы - карандаш, висящий в воздухе МКС, стремится попасть в приливный захват из-за градиента гравитации, и вращается как качели туда-сюда.
> Но есть же и какие-то другие силы?
Например анизотропия, связанная с направлением орбиты. Если поместить плавать два шарика последовательно по направлению орбиты, ничего не произойдет. Если же повесить их параллельно - один чуть левее, другой чуть правее линии орбиты - они будут сходиться и расходиться каждые полвитка. Потому что они на самом деле летят каждый по своей орбите, а она проходит через центр масс, и плоскости близких орбит пересекаются. Собственно поэтому в правилах ВКД строго запрещено отбрасывать предметы "вбок" станции, если это требуется по плану - т.к. уже через полвитка предмет прилетит обратно с той же скоростью. Отбрасывают только назад (в ретрогрейд).
Аноним 13/05/15 Срд 10:33:21 #117 №178808 
>>178801
Гугли что такое масса.
Аноним 13/05/15 Срд 11:11:34 #118 №178813 
>>178801
1) Совершенно постыдный момент фильма, пустивший по пизде физику ради драмы. После того, как они, зацепившись за трос, отлетели достаточно далеко, чтобы трос натянулся, их должно было дёрнуть обратно к МКС.
Смотри момент в начале фильма, когда они ещё на шаттле, который разлетается под ударами обломков. Там мужику на заднем плане прилетает осколком в шлем, его толкает, затем страховочный трос натягивается и дёргает его обратно.
Аноним 13/05/15 Срд 17:07:55 #119 №178890 
Как в твёрдотопливных МБР регулируется дальность полёта? Ведь двигатель фактически неуправляем. В голову приходит только увеличение/уменьшение крутизны траектории, но это жи куча ограничений.
В /wm/ быдло малограмотное, поэтому сюда.
Аноним 13/05/15 Срд 17:42:13 #120 №178902 
>>178890
Как будто здесь кто-то смыслит в МБР.
>В голову приходит только увеличение/уменьшение крутизны траектории
Ну они могут летать в некоторых пределах - есть оптимальные траектории, есть пологие, есть высокие (depressed vs lofted). Ограничений соответственно тоже дохуя как я понимаю - нагрев БЧ, перегрузки, видимость для радаров, достижимость для ПРО и т.п.

Алсо, про укладку ТТРД поясняли в прошлом треде >>170012 >>170039, а я дополню что ТТРД бывают еще сегментированными с заслонками между сегментами, или вообще раздельным их поджигом, так что в ограниченных пределах тяга таки регулироваться может, хотя это неоптимальное решение. Сомневаюсь что такое применяется на МБР, правда.
Аноним 13/05/15 Срд 18:16:16 #121 №178909 
>>178890
Тягу твердотопливных хуень можно регулировать, в некоторых пределах.

https://youtu.be/F_gpGbwfwFc
Аноним 13/05/15 Срд 18:54:23 #122 №178928 
Поясните, космическая промышленность Украины - все?
Аноним 13/05/15 Срд 19:45:44 #123 №178960 
>>178801>>178813

Кстати, этот момент в Гравитации был-таки научным, и мне сразу показался "естественным".

Дело не в том, что Райан (тян) не хватает сил - просто после захвата Ковальски двое астронавтов продолжают удаляться от станции на быстро разматывающихся/натягивающихся стропах, который недостаточно устойчиво зафиксированы на ноге Райан, чтобы надежно погасить их общий импульс. Это очевидно в кадрах снятых с расстояния. В кадрах "сблизи" кажется, что они неподвижны, а Ковальски просто тянет какая-то ебическая сила. На самом деле они движутся, и Ковальски просто понял, что с их общим импульсом (у Ковальски масса заметно больше т.к. MMU) Райан не удержится на стропах.
Аноним 13/05/15 Срд 20:15:20 #124 №178975 
>>178960
>Это очевидно в кадрах снятых с расстояния
Пили эти самые кадры в вебм, или ссылку на тытруб. Я, конечно, мог и подзабыть, но никакого вращения там ни в каком виде не помню.
Аноним 13/05/15 Срд 20:20:09 #125 №178979 
>>178890
Все твердотопливные МБР содержат третью жидкотопливную ступень.
мимо быдло из /wm
Аноним 13/05/15 Срд 20:25:47 #126 №178986 
14315379477370.jpg
Кто помнит?
Аноним 13/05/15 Срд 20:29:23 #127 №178990 
>>178986
Зачем это помнить
Нифига смешного или остроумного
Аноним 13/05/15 Срд 20:32:31 #128 №178993 
>>178990
А вот и ньюфаги подкатили.
Аноним 13/05/15 Срд 20:33:31 #129 №178994 
>>178928
У южмаша еще остались комплектующие для 3х Зенитов. На 2015 год запланирован запуск с Байконура. Если снимут эмбарго на техническое сотрудничество с Российскими предприятиями, то может быть еще есть шанс и дальше делать ракеты, но перспективы очень туманны.. Короче про такую ракету как Зенит можно забыть навсегда.

>промышленность Украины - все?
Да.
Аноним 13/05/15 Срд 20:38:50 #130 №178996 
>>178994
А куда специалисты уйдут?
На грядки сало растить?
Аноним 13/05/15 Срд 20:53:12 #131 №179007 
Есть 59 широта.
В какое время ночью будет самая тьма?
В мае.
Везде есть общеизвестные данные, типа восхода и заката, время начала и конца различных сумерек.
Но нету «Вот вообще темно, и сейчас ночь будет переходить в рассвет»
Или я просто не знаю термина.
Аноним 13/05/15 Срд 21:36:17 #132 №179016 
>>178986
Битард, плиз. Тут серьезные господа сидят, им не до щитпостинга в компании ололокающих школьников.
Аноним 13/05/15 Срд 22:30:32 #133 №179047 
>>178975
ну ты ёбу-то не давай, я про вращение ничего не писал
Аноним 13/05/15 Срд 22:35:03 #134 №179050 
>>179007
Между восходом и закатом, очевидно же.
Аноним 13/05/15 Срд 22:57:06 #135 №179061 
Пацаны, а если черную дыру ничего не может покинуть, ни вещество, ни свет, ни аллах, то как же она испаряется? Или врет Хокинг? Существует ли хоть одна наблюдаемая черная дыра? То есть не сама, а допустим аккреционный диск.
Аноним 13/05/15 Срд 23:39:36 #136 №179080 
Как объясняется эффект гравитационного линзирования?Почему свет притягивается массивным телом?Ведь у фотонов нет массы, не говоря уже про электромагнитные волны.
Аноним 13/05/15 Срд 23:49:34 #137 №179087 
14315501742900.jpg
Вопросик из стареньких чёт всё не отвечают - вота мы видим Здеся скопление галлактик, ну, разлетается. А вот Тута - поближе и кучнее - неразлетается. Я грю - а чо так? А БВшники грят - ну, тип, Здеся расширение пространства, все дела, вот и разлетаются в этом пространстве ну тип того. А Тута другое дело - близко они скопления, за друг дружку гроветаццей держатся и неразлетаются. А я такой опять - а чё, а тип раньше, недалеко от БВ как вы грите, все были близко, все держались гроветаццей, чё такого произошло чё они прощёлкали момент и таки стали разлетаться - гравитацца кончилась?
Аноним 14/05/15 Чтв 00:09:37 #138 №179104 
>>179080
E=mc^2
Аноним 14/05/15 Чтв 00:10:07 #139 №179105 
>>179104
Но массы же нет
Аноним 14/05/15 Чтв 00:12:17 #140 №179107 
>>179061
Наблюдаемых весьма не мало, тот же стрелец альфа
Это видео может ответить на часть твоих вопросов
https://youtu.be/2zfPk-Iw5y0
Аноним 14/05/15 Чтв 00:13:10 #141 №179108 
>>179105
Есть энергия и скорость света
И энергия не равна нулю
Вся суть в том, что нулевая - масса покоя
Аноним 14/05/15 Чтв 00:25:50 #142 №179117 
>>179108
>масса покоя

То есть хочете сказать, что в стекле, скорость света в котором меньше в 2 раза, масса фотонов будет меньше?
Аноним 14/05/15 Чтв 00:27:56 #143 №179119 
>>179117
Ты чё несёшь?
Фотоны всегда движутся со скоростью света, другое дело что они не всегда летят в одном направлении
Аноним 14/05/15 Чтв 00:53:29 #144 №179142 
14315540090650.jpg
14315540090671.jpg
>>178890
Последняя ступень ориентируется по звездам и тормозит, или ускоряется, дальше блок разведения наводит ГЧ на цель.
Аноним 14/05/15 Чтв 01:03:38 #145 №179153 
В другом треде не ответили, может быть здесь ответят?
Недавно заинтересовал вопрос зарождения Вселенной и того что с ней происходит.
Узнал о законе Хаббла, Большом взрыве и инфляционной модели Вселенной. Все они говорят о расширении вселенной. Закон Хаббла основывается на том, что при удалении галактик у них наблюдается красное смещение в спектре, подобно эффекту Доплера. Однако далее я узнал о таком понятии, как гравитационное красное смещение, т. е. возникновение красного смещения в связи с прохождением света через сильные гравитационные поля (как раз от звезд, галактик). Из-за данного эффекта ставится под сомнение верность закона Хаббла и, соответственно, теории Большого взрыва и инфляционного расширения Вселенной.
Где все-таки правда, и где ошибки? Какие теории на данный момент являются наиболее правдоподобными?
Аноним 14/05/15 Чтв 01:05:57 #146 №179155 
>>179153
Теория большого эфира и суперструнметрия
Аноним 14/05/15 Чтв 01:08:23 #147 №179156 
>>178996
Многим уже предложили переехать в соседнюю страну. Там всех годных спецов знают пофамильно. Остальные не знаю куда.
>>179007
http://suncalc.net/
>>179087
>чёт всё не отвечают
Давно все ответили.
>все были близко, все держались гроветаццей
Кто все блядь? В первые полмиллиарда лет и галактик не было - первые звезды только-только появлялись.
>>179153
http://www.modcos.com/articles.php?id=120
>>179153
>ставится под сомнение верность закона Хаббла и, соответственно, теории Большого взрыва и инфляционного расширения Вселенной.
Нехуя, гравитационное красное смещение касается каких-то отдельных, частных случаев, а не всех фотонов, со всех сторон.
Аноним 14/05/15 Чтв 01:17:51 #148 №179163 
>>179156
>Кто все блядь? В первые полмиллиарда лет и галактик не было
А, ну раз ГАЛЛАКТИК НЕ БЫЛО, то конечно, всё нормально, МАССА же только в галлактиках бывает, она только в них умеет гравитационно взаимодействовать. Хуле умничаешь, тупая мразь?

Будем ждать ответа более просвящённого и не мрази.
Аноним 14/05/15 Чтв 01:23:13 #149 №179166 
>>179163
Дебил, иди нахуй отсюда.
Аноним 14/05/15 Чтв 01:33:35 #150 №179174 
>>179107
Ага, спасибо!
Аноним 14/05/15 Чтв 01:44:54 #151 №179178 
Поясните за эпоху вечной тьмы, почему частицы не смогут собираться в атомы?
Аноним 14/05/15 Чтв 01:54:37 #152 №179179 
>>179178
Скорость и температура. При высокой температуре частицы очень быстро движутся, обладая высокой энергией. На такой скорости любое столкновение частиц отрывает электрон от протона.
Аноним 14/05/15 Чтв 02:00:19 #153 №179180 
>>179179
А с чего кстати взяли, что протоны распадутся?
Аноним 14/05/15 Чтв 02:03:59 #154 №179181 
1. Могла ли сингулярность взорваться иначе, чтобы в нашей вселенной были другие физические законы?
2. Возможны ли космические аппараты на основе биотехнологий? Какие профиты?
3. Каких технологий не хватает чтобы орбитальные полёты в космос стали доступны например по цене 10 тыс $ за билет?
Аноним 14/05/15 Чтв 02:10:03 #155 №179187 
>>179180
>А с чего кстати взяли, что протоны распадутся?
Нет никаких оснований, или предпосылок, запрещающий распад протона. Поэтому сделали вывод, что должен распадаться.
>>179181
>Могла ли сингулярность взорваться иначе, чтобы в нашей вселенной были другие физические законы?
Могла.
>на основе биотехнологий?
Корабль - жЫвотное штоле, как в LEXX?
>по цене 10 тыс $ за билет?
Пузырь Альбукерке разве только. И то при условии халявной энергии в больших количествах.
Аноним 14/05/15 Чтв 02:17:05 #156 №179189 
>>179187
> >Могла ли сингулярность взорваться иначе, чтобы в нашей вселенной были другие физические законы?
> Могла.
Известно что для этого нужно было
> Корабль - жЫвотное штоле, как в LEXX?
Лол, да. Как раз сейчас смотрю.
> Пузырь Альбукерке разве только. И то при условии халявной энергии в больших количествах.
Очень жаль
Аноним 14/05/15 Чтв 02:24:50 #157 №179190 
>>179189
>Известно что для этого нужно было
Нет, про этот период вообще ничего не известно, одни гипотезы. Почему Вселенная именно такая, а никакая другая - вопрос философский больше, потому что человеческую физику туда не приплетешь.
>Как раз сейчас смотрю.
Нет, жуков-звездолетов, скорее всего не будет. Жесткая радиация делает много "дырок" в ДНК, от чего мутации и всякие сбои.
>Очень жаль
Ну может быть когда-нибудь людишки разберутся с гравитацией, ну гравитонов наловят, или волны поймают, или квантовую гравитацию за яйца поймают, и что-нибудь придумают, но пока особых поводов для оптимизма чего-то нет.

Аноним 14/05/15 Чтв 02:45:12 #158 №179192 
>>179190
Ну хз, тихоходки например выдерживают ебические дозы излучения, живут сутками в космосе.
Например когда начнут ковыряться в геноме человека можно сделать космонавтов стойкими к облучению для длительных полётов, а вместо веса биозащиты взять больше топлива или груза.
Аноним 14/05/15 Чтв 03:20:08 #159 №179195 
>>179192
>можно сделать космонавтов стойкими к облучению

И они станут не очень умными тихоходками.
Аноним 14/05/15 Чтв 03:21:49 #160 №179196 
>>179156
>галактик не было

Масса была?
Аноним 14/05/15 Чтв 03:23:51 #161 №179197 
>>179195
С чего вдруг
Аноним 14/05/15 Чтв 03:32:19 #162 №179198 
>>179197
Тихоходки стойкие наверно не потому, что у них хитрая таблетка в животе или жидкость в крови, а потому что они ТАКИЕ вот целиком и полностью. Вряд ли есть чудесный витамин, который всего-то нужно добавить в космонавта и получить супермена. Нужно менять всё.
Аноним 14/05/15 Чтв 03:48:19 #163 №179199 
>>179198
Ну можно и поменять
Аноним 14/05/15 Чтв 03:59:34 #164 №179200 
>>179197
Насекомые устойчивее к радиации, а такие организмы, как тихоходки, еще более устойчивы, чем примитивнее и меньше организм, тем меньше негативных факторов на него действует. Они быстрее размножаются и в большем количестве, так что человек получает одного дебила за год, и максимум 5-7 детей за жизнь, а мелочь воспроизводится сотнями. Так что этот анон прав >>179198 хочешь получить 500000 рентген и шанс на выживание - становись тихоходкой целиком.
Аноним 14/05/15 Чтв 04:00:16 #165 №179201 
14315652163920.jpg
>>179199
Да как нехуй.
Аноним 14/05/15 Чтв 04:09:34 #166 №179202 
>>179200
Но можно ли стать большой тихоходкой с человеческими мозгами?
Аноним 14/05/15 Чтв 04:22:20 #167 №179203 
>>179202
Как бы тебе объяснить.
>Что проще, то устойчивее.
Тихоходка простая и неприхотливая. Мозг человека - одна из самых сложных и капризных поебеней, созданных природой. Улавливаешь?
Аноним 14/05/15 Чтв 04:27:07 #168 №179204 
>>178325
Это очень милый пост :3
Аноним 14/05/15 Чтв 04:40:53 #169 №179205 
>>179007
Термин - терминатор, лол.
Аноним 14/05/15 Чтв 04:47:21 #170 №179206 
>>179198
Обклеить космонавта тихоходками?
Аноним 14/05/15 Чтв 04:48:12 #171 №179207 
Вы все дебилы. Если у жука звездолета панцирь тупо НЕ БУДЕТ пропускать радиацию, то норм он летать будет.
Аноним 14/05/15 Чтв 04:49:00 #172 №179208 
>>179201
Проиграл.
Аноним 14/05/15 Чтв 05:25:44 #173 №179209 
>>179203
Алсо, биохимия у человека будет посложнее тихоходской, а мозг без родной биохимии врежет дуба еще до того, как успеет осознать себя тихоходкой.
>>179207
>панцирь тупо НЕ БУДЕТ пропускать радиацию
И что же это за панцирь, который не будет пропускать радиацию? 10 метров хитина? Жрать оно будет астероиды? А двигаться?
Аноним 14/05/15 Чтв 08:29:12 #174 №179213 
>>176361
Хорошо, что нового появилось и как оно ломает старые теории?
Аноним 14/05/15 Чтв 09:35:26 #175 №179225 
14315853266820.jpg
>>179187
>Корабль - жЫвотное штоле, как в LEXX?
А я вспомнил о рейфовских кораблях из Атлантиды.
Аноним 14/05/15 Чтв 10:17:38 #176 №179229 
>>179209
Да хоть из обеднённого урана
Аноним 14/05/15 Чтв 11:23:02 #177 №179235 
>>179225
Еще есть мойа из фарскэйпа.
Аноним 14/05/15 Чтв 13:38:54 #178 №179249 
Если скорость света есть максимально допустимая, то как сам свет/фотон движется с такой скоростью, как на него действуют релятивистские эффекты, масса же есть? Таким образом, относительно фотона он летит от Солнце к Земле за бсконечно малое время? Где я наебался?
Аноним 14/05/15 Чтв 14:15:36 #179 №179252 
>>176350
Если взять самый большой телескоп за миллиард из действующих, направить его на Марс, картинку с какой разрешающей способностью можно получить? метров на пиксель
Аноним 14/05/15 Чтв 14:20:28 #180 №179254 
> Солнечное вещество размером с булавочную головку, помещенное в атмосферу нашей планеты, начнет с невероятной скоростью поглощать кислород и за доли секунд уничтожит все живое в радиусе 160 километров.
Не пиздеж?
Аноним 14/05/15 Чтв 14:39:06 #181 №179259 
>>179254
Тупопёздный пиздеж. Бздюхнет как петарда и все. Энергии ни на что не хватит.
Аноним 14/05/15 Чтв 14:39:22 #182 №179260 
>>179080
>Ведь у фотонов нет массы
Пруф?
>Как объясняется эффект гравитационного линзирования?
Изменение коэффициента преломления среды вакуума под действием различных причин.
Аноним 14/05/15 Чтв 14:45:50 #183 №179263 
>>179153
>Где все-таки правда, и где ошибки?
БВ наебалово. КС происходит от потери энергии фотонов при долгом движении через среду вакуума. Вклад Доплера мал, т.к. собственные движения голактек в среднем одинаковы. Среда вакуума возле массивных объектов имеет чуть другие свойства, поэтому КС там протекает с другой скоростью.
>Какие теории на данный момент являются наиболее правдоподобными?
Эфирные.
Аноним 14/05/15 Чтв 14:59:15 #184 №179265 
>>179080
Свет не притягивается - это пространство искривляется под действием массы, а свет летит прямо, как и летел.
Аноним 14/05/15 Чтв 15:05:20 #185 №179266 
Что было до большого взрыва? С пруфами.
Аноним 14/05/15 Чтв 15:12:25 #186 №179268 
14316055456170.jpg
14316055456191.gif
>>179266
Он. Пруфы - пикрелейтед.
Аноним 14/05/15 Чтв 15:51:05 #187 №179276 
>>179268
>SH
Но это всего лишь большие относительно пустые от галактик области за сферой Хаббла.
Аноним 14/05/15 Чтв 15:58:39 #188 №179280 

>>179266
>Что было до большого взрыва? С пруфами.
42
Аноним 14/05/15 Чтв 16:01:05 #189 №179282 
>>179263
Ты ещё не во всех тредах успел своим кефиром насрать, слабоумный?
Аноним 14/05/15 Чтв 16:20:58 #190 №179290 
>>179282
Я несу в твой мозг просвещение, наивный мой.
Аноним 14/05/15 Чтв 16:43:18 #191 №179306 
>>179268
Что на втором скрине?
Аноним 14/05/15 Чтв 16:43:40 #192 №179307 
>>179290
Свой мозг просвети, желательно рентгеном, или МРТ сделай, у тебя явно какая-то патология.
Аноним 14/05/15 Чтв 17:38:10 #193 №179343 
>>179282
Смотрите, маняучоный заголосил!
sageАноним 14/05/15 Чтв 17:41:14 #194 №179346 
>>179343
Слышь, петух, ты не думай что тут с спейсач одни слабосильные, энурезников тоже хватает.
Обоссал кефирщика.
Аноним 14/05/15 Чтв 17:43:54 #195 №179348 
>>179346
Думаю, что обоссать кефирщика - это священный долг каждого спейсачера. Тут даже самые интеллигентные и стеснительные должны поднапрячься и поучаствовать в этом процессе.
Аноним 14/05/15 Чтв 17:48:23 #196 №179351 
>>179306
Афтограф Создателя на карте вселенной, лол
карта распределения плотности реликтового микроволнового излучения
sageАноним 14/05/15 Чтв 17:50:27 #197 №179352 
Кефироблядь залетела, хоть какое развлечение. Присоединяю свою тугую струю.
Аноним 14/05/15 Чтв 17:56:42 #198 №179354 
>>179348
+15, good goy.
Аноним 14/05/15 Чтв 18:50:19 #199 №179375 
14316186198900.jpg
Каков гипотетический удельный импульс трехкомпонентного ЖРД на компонентах Фтор-Керосин-Водород??
Аноним 14/05/15 Чтв 20:27:56 #200 №179407 
>>179375
Он просто охуителен
Аноним 14/05/15 Чтв 20:33:58 #201 №179409 
>>179375
https://ru.m.wikipedia.org/wiki/РД-701
А так, говорят что 550
Аноним 14/05/15 Чтв 20:39:13 #202 №179412 
>>179375
>трехкомпонентного ЖРД на компонентах Фтор-Керосин-Водород
Это как? Они все вместе используются в одной реакции? Если все же попеременно, то не имеет смысла говорить об удельном импульсе всей системы.
Аноним 14/05/15 Чтв 20:44:59 #203 №179413 
>>179412
Вики в помощь
Аноним 14/05/15 Чтв 20:52:50 #204 №179417 
>>179413
Зачем? Там описано, как использовать фтор, керосин и водород в одной реакции? Или ты про РД-701? Ну он двухрежимный и есть. Какой у двухрежимного движка может быть общий удельный импульс? Он работает сначала так, а потом эдак. Как SABRE какой-нибудь.
Аноним 14/05/15 Чтв 20:56:54 #205 №179419 
>>179417
Я так понимаю что продукты реакции двух этих веществ потом дожигаются с использованием третьего
Аноним 14/05/15 Чтв 20:59:02 #206 №179421 
>>179419
Так это возможно только с легкими металлами, литий тот же. Они так и называются, металлосодержащие топлива. С керосином, выбрасывая водород - не выйдет.
Аноним 14/05/15 Чтв 21:23:20 #207 №179430 
>>176350
Сап, Двач. Я — космонавт. У меня скоро первый полёт. И я никак не могу определиться, какую молитву читать во время взлёта: канон покаянный ко Господу нашему Иисусу Христу или же молитва перед отправлением в воздушное путешествие?
Срочно.
Аноним 14/05/15 Чтв 21:39:05 #208 №179440 
>>179430
Главное колесо обоссы и молись хоть Ахура Мазде - все будет норм
Аноним 14/05/15 Чтв 21:40:35 #209 №179441 
>>179417
>РД-701
Сперва работая на первом режиме в камеру сгорания подается 3 компонента, при этом масовая часть водорода не превышает 8% и за счет этого достигается удельный импульс в 410 единиц у земли и 200 тонн тяги.

Во втором цикле по выработке керосина двигатель переходит на второй Кислород-Водородный режим при этом удельный импульс равен 460 еденицам, а тяга снижается до 80 тонн.
Аноним 14/05/15 Чтв 21:46:12 #210 №179443 
>>179441
Эта схема позволяет использовать все преимущества керосина(высокая плотность горючего) и водорода (высокий удельный импульс) при этом позволяет избавится от такого недостатка водородного топлива как огромный криогенный бак.
Аноним 14/05/15 Чтв 22:04:15 #211 №179449 
>>179181
3. Технологий постройки гигантских йоб вроде пусковой петли и космолифта. Тут и геополитика мешает, терроризм, аллах.
Аноним 14/05/15 Чтв 22:07:37 #212 №179453 
>>179254
Жечь будет только при условии, что не будет охлаждаться, и то рентгеном и уф наверно.
Аноним 14/05/15 Чтв 22:41:28 #213 №179460 
>>179443
Это скорее ухудшение водородного движка, чем улучшение керосинового. За что еще и придется платить массой и сложностью. И вообще, РД-701 это гипотетическая вундервафля, никогда не воплощавшаяся в железе. Как и РД-0750, но там вроде какие-то работы были, прежде чем дропнули этот омск.
> от такого недостатка водородного топлива как огромный криогенный бак
Недостаток надуманный, ибо размер растет пропорционально кубическому корню объема. Delta IV например прекрасно летает на водороде в первой ступени и благодаря высокому импульсу она может даже в режиме SSTO вытащить небольшую нагрузку теоретически, несмотря на диаметр бака и прочую хуйню. Водород в движке 1 ступени сложнее юзать по куче других причин - плохое массовое совершенство, плохо масштабируется, тяга меньше (и гравипотери вначале траектории выше), и т.п.

А в такой смесевой йобе водород и так есть, платить за это придется, и в то же время полного профита с него не выходит.

>>179453
Не будет охлаждаться только если не будет расширяться, и если взять материал ядра (которое собственно горит), и его там с гулькин нос. И то погаснет все равно без окружающего вещества. А так-то оно расширится и остынет моментально. Ну будет у тебя не одна булавка, а 150. Максимум хлопнет чутка.
Аноним 14/05/15 Чтв 23:07:40 #214 №179463 
14316340610220.jpg
14316340610331.jpg
>>179460
>РД-701 это гипотетическая вундервафля, никогда не воплощавшаяся в железе.
Были полноразмерные технологические макеты и есть вся необходимая тех документация для этого двигателя
Были полномасштабные технологические макеты и вся нужная тех документация для РД-701.
>9 августа 1994г Проведено первое огневое испытание экспериментального трехкомпонентного ЖРД на стенде НИИХиммаш в Сергиевом Посаде с целью подтверждения концепции трехкомпонентных двухрежимных ЖРД РД-701 и РД-704.

>РД-0750, но там вроде какие-то работы были, прежде чем дропнули этот омск.
Его в 98 году даже успешно прожигали подтвердив заявленные характеристики.
Ну а дропнули его не из за технологических проблем, а банально не оказалось денег на разработку, да и задачь под него не нашли.
Аноним 14/05/15 Чтв 23:51:27 #215 №179468 
>>179463
>9 августа 1994г Проведено первое огневое испытание экспериментального трехкомпонентного ЖРД на стенде НИИХиммаш в Сергиевом Посаде с целью подтверждения концепции трехкомпонентных двухрежимных ЖРД РД-701 и РД-704
А вот люди в теме на НК утверждают, что все это брехня пиар-отделов и заинтересованных в распиле сторон, и процитированное было лишь испытанием камеры сгорания (без ТНА), да и полноразмерного ничего там не было (10т тяги).
>Я разговаривал с людьми с Энергомаша.
>Это хорошо известный макет который много раз кочевал по выставкам. Первый раз если не ошибаюсь я его увидел на выставке "К звёздам-91".
>Никаких реальных или уменьшеных ЖРД нигде не испытывалось и даже не было изготовлено. Проводились опыты с маленькими опытными камерами сгорания со стендовой подачей компонентов.
В любом случае, это беспредметный разговор, т.к. инфа про них везде довольно сомнительная и противоречивая. Про РД-0750 уже другой разговор.
Аноним 15/05/15 Птн 00:29:59 #216 №179478 
Спейсач, поясни пожалуйста. Я тут посмотрел Интерстеллар, и у меня возникли вопросы:
1. Когда корабль пролетал на фоне Черной дыры, мне показалась она слишком маленькой, т.е. разве черный дыры не должны быть неизмеримо большими по сравнению с космическим кораблем? А тут кажется словно на орбиту нашей Ланы вышел.
2. Разве может косм. корабль находится на таком близком расстоянии от ЧД и оставаться невредимым? А как же радиация?
3. Существуют ли на самом деле системы подобно Гаргантюа? Разве планеты не будут утянуты мощной гравитацией ЧД?
Аноним 15/05/15 Птн 00:43:10 #217 №179483 
>>179209
че это тебе 10 метров хитина не понравилось ?
двигаться, ну ты загнул - мы же про корабель так ? тяга же

но так в принципе у живого корабля особенных бонусов то и нет, за исключением переноса паттерна - я люблю собаку-кошку-бегемота-лошадь
схемы пизданулся на планету, раскусил полый зуб плюнул на субстрат и начал выращивать космодром (и живой корабль тоже) - могут быть реализованны и более прямыми методами. в том числе с конечным итогом в виде сферического коня, если уж кровь требует когото мыть поощрять и ебать-наказывать.
Аноним 15/05/15 Птн 00:47:14 #218 №179485 
>>179249
>Где я наебался?
>Таким образом
вот здесь, в дано у тебя нихуя
Аноним 15/05/15 Птн 01:20:51 #219 №179498 
Генетическое манипулирование интересня тема кстати
Если бы человечество не потратил тысячи лет на техногенное развитие мир был бы весьма забавен
Вроде что-то такое было то ли у Гаррисона то ли у тердлава про альтернативную эволюцию с рептилоидами которые торчали по биотехнологиям
Эдем или типа того называлась
Надо перечитать
Аноним 15/05/15 Птн 01:26:11 #220 №179499 
>>179498
Если бы человечество не потратило тысячи лет на развитие технологий, мы бы всё ещё пиздили друг друга палками и ни про какие гены не подозревали.
Аноним 15/05/15 Птн 01:36:20 #221 №179504 
>>179499
Вероятно интуитивное развитие генетики
Аноним 15/05/15 Птн 02:24:17 #222 №179520 
>>179252
бамп
Аноним 15/05/15 Птн 03:35:15 #223 №179523 
Поясните за луну - зачем нужен такой уродливый спутник? Он же выглядит как говно
Аноним 15/05/15 Птн 03:40:44 #224 №179524 
>>179523
>Поясните за горы, зачем нужны эти непонятные хуйни? Они же выглядят как говно
Аноним 15/05/15 Птн 04:34:26 #225 №179527 
>>179478
>Когда корабль пролетал на фоне Черной дыры, мне показалась она слишком маленькой
Тебе показалось: угловые размеры не совпадают с реальными. Например когда показывали планету Миллер, то было вполне очевидно, что ЧД неебического размера. Скриншот можешь загуглить.
2.Может,
>А как же радиация?
Есть концепция холодных аккреционных дисков, в системе Гаргантюа давно жрать нечего, это старая система, так что и газ, закрученный в диск не излучает очень сильно, так как его очень мало. Он разогрет до высоких температур, в 3-5 тысяч K, но не до сверхвысоких, как в активных ядрах. Короткое пребывание около такого диска не отразится слишком серьезно на здоровье астронавтов. Пример таких спокойных дыр и холодных дисков ИРЛ: Стрелец А* и Лебедь-Х1.
3. Стандартное заблуждение о том, что ЧД засасывают все вокруг. ЧД-это просто очень массивный объект, Солнце тоже очень массивный объект, по сравнению с окружающими говнами, но нас вместе с Земляшкой в Солнце не засосало.
>Существуют ли на самом деле системы подобно Гаргантюа
Существование планетных систем вроде Гаргантюа маловероятно, планеты вряд ли бы образовались рядом с ЧД, там еще недалеко нейтронная звезда, которую вскользь упоминают в фильме. Скорее всего планеты захвачены извне, но тогда бы орбиты вряд ли были бы близким к круговым. Хотя вот Кип Торн в своей The Science of Interstellar пишет, что Миллер образовалась и существует на своем месте, а Манн - захваченная планета. В любом случае, даже если такая система и маловероятна, то учитывая размеры Вселенной, она просто неизбежно существует.
Аноним 15/05/15 Птн 04:37:46 #226 №179528 
>>179523
Поясните за этого сосачера, зачем нужен этот непонятный хуй? Он же выглядят как говно Уверен, что пахнет так же.
Аноним 15/05/15 Птн 16:44:45 #227 №179627 
>>179523
Да, выглядит как говно, но щито поделать, другого не завезли. Будем с этим мучаться
Аноним 15/05/15 Птн 16:56:58 #228 №179628 
>>178960
А вот и нифига. Прекрасно видно 1:16, как их тянет какая-то ебическая сила, которая отпускает, как только Ковальски отцепляется. Хотя перед этим Райан дёрнуло стропами назад.
Аноним 15/05/15 Птн 17:06:36 #229 №179629 
>>179249
Фотон - гамма-квант электромагнитного излучения, у него нет массы. Он существует только пока двигается.
Аноним 15/05/15 Птн 18:00:16 #230 №179638 
>>179629
Строго говоря, есть ли масса у фотона, или нет - вопрос пока открытый и до конца не решенный.
Аноним 15/05/15 Птн 18:13:22 #231 №179642 
>>179638
Если принять энергию фотона за его массу, то да, можно считать, что она есть. Но КРАЙНЕ МАЛА.
Аноним 15/05/15 Птн 19:10:31 #232 №179648 
>>179642
Нет, я не об этом.
http://www.nature.com/news/big-bang-light-reveals-minimum-lifetime-of-photons-1.13474
http://physics.aps.org/synopsis-for/10.1103/PhysRevLett.111.021801
Аноним 15/05/15 Птн 19:20:46 #233 №179650 
>>179648
Я читал то же самое, но по-русски. Видимо, мы пришли к разным выводам или же говорим о разных вещах.
Если я правильно понимаю, то масса фотона в любом случае будет вычисляться через энергию.
Аноним 15/05/15 Птн 19:26:50 #234 №179651 
>>179650
Масса любой частицы привычно выражают в электронвольтах, так удобнее, но это не значит, что у нейтрино, например, нет массы, а есть только энергия. Тоже самое и с фотонами. Считается, что у него нет собственной массы, т.н. "массы покоя", но есть подозрения, что она просто очень мала. Пока на уровне гипотез, конечно.
Аноним 15/05/15 Птн 19:34:10 #235 №179652 
>>179651
Значит, я всё правильно понял :)
Аноним 15/05/15 Птн 19:37:26 #236 №179653 
>>179652
В некоторой степени. Осталось понять остальное. :3
Аноним 15/05/15 Птн 21:11:29 #237 №179673 
Как летает эта газонокосилка?
https://www.youtube.com/watch?v=kNWfqVWC2KI
Аноним 15/05/15 Птн 22:40:26 #238 №179680 
14317188265570.jpg
>>179673
Это очень аэродинамическая газонокосилка.

Аноним 16/05/15 Суб 00:08:31 #239 №179722 
14317241119670.jpg
14317241119691.jpg
>>179673
>Как летает эта газонокосилка?
Как обычный самолет. У нее пропеллер и аэродинамические поверхности из легкого материала. По сути дела - это летающее крыло, задрапированное под газонокосилку.
Аноним 16/05/15 Суб 00:43:17 #240 №179744 
>>177291
Простите, а на русском подобного нет?
Аноним 16/05/15 Суб 01:01:43 #241 №179751 
Допустим, наша вселенная не одна а их несколько. Как должен происходить переход из одной в другую? В каком пространстве они находятся? Может есть какие-нибудь теории?
Аноним 16/05/15 Суб 01:05:14 #242 №179752 
>>179673
Это что. Ты еще летающий танк не видел.
https://www.youtube.com/watch?v=jKUdvs-atJI
Аноним 16/05/15 Суб 01:13:41 #243 №179754 
>>179751
>В каком пространстве они находятся?
>Может есть какие-нибудь теории?
Нет никаких теорий, заслуживающих внимания, в самой концепции мультивселенной проблема - Поппер от нее в могиле немного ворочается. Можешь себе нафантазировать что угодно - проверить это никак не представляется возможным.
Аноним 16/05/15 Суб 01:53:23 #244 №179764 
Может ли земляшка "захватить" мимопролетающий астероид и сделать его своим спутником?
Аноним 16/05/15 Суб 04:01:12 #245 №179780 
Могут ли в нашей вселенной существовать дву- или более чем трехмерные области пространства?
Аноним 16/05/15 Суб 08:22:12 #246 №179796 
>>176350
Почему еще не запилили нормальную связб с ниской задержкой в пределах солнечной системы хотя бы? Ведб досихпор юзают древние технологии дидов для связи. Стали юзать цифровые данные, появились более мощные передатчики, а суть та же, древнее как говно мамонта радиосвязь. Нужна как минимум реалтаймовая (с очень низкой задержкой) связь до Марса, чтоб запилить нормальный межпланетный интернет.
Аноним 16/05/15 Суб 09:13:58 #247 №179797 
>>179796
Потому что скорость света ~ 300000 км/с
учись в физику, школьник
Аноним 16/05/15 Суб 09:15:29 #248 №179799 
>>179797
можно запилить сверхсветовой сигнал.
Аноним 16/05/15 Суб 09:20:03 #249 №179800 
>>179799
На каких технологиях?
Аноним 16/05/15 Суб 09:21:52 #250 №179802 
>>179799
Дед Эйнштейн щас встанет из могилы и запилит тебе в табло. "Сигнал" выше скорости света разогнаться не может. Никогда. Вообще. Совсем.
Аноним 16/05/15 Суб 09:43:14 #251 №179805 
>>179802
ну это пока
Аноним 16/05/15 Суб 10:23:56 #252 №179816 
14317610361900.jpg
>>179805
Пояснил мне за сверзсветовую связь, быстраблять!!!
Аноним 16/05/15 Суб 10:34:40 #253 №179818 
>>179816
Вангую, что он сейчас заикнётся про квантовую спутанность.
Аноним 16/05/15 Суб 10:50:21 #254 №179821 
>>179818
Или даже про кротовые норы
Аноним 16/05/15 Суб 14:03:16 #255 №179861 
>>179764
Конечно может. Вопрос - почему этого еще не случилось?
sageАноним 16/05/15 Суб 14:24:06 #256 №179874 
>>179861
чтобы захватить астероид, он должен попасть в атмосферу, и затем ускорится, чтобы орбита не была в атмосфере. А еще ему надо умудриться не сгореть, короче анриал.
sageАноним 16/05/15 Суб 14:25:30 #257 №179875 
>>179764
Такой как в ксп врятли.
Аноним 16/05/15 Суб 14:40:22 #258 №179879 
>>179764
Может, Марс таких аж два захватил (скорее всего). Но для этого нужна череда ебических совпадений. В системе двух тел захват невозможен, нужно третье, которое поможет погасить астероиду часть энергии. Например Луна или второй астероид в двойной системе (один захватывается, другой улетает). Либо разрушение объекта при прохождении внутри полости Роша или в результате столкновения с другим (куски остаются на орбите).
Аноним 16/05/15 Суб 14:53:26 #259 №179887 
14317772065980.gif
>>179764
Аноним 16/05/15 Суб 16:07:51 #260 №179929 
>>179527
>Тебе показалось
Тем не менее обогнули ее они пиздец как быстро.
Аноним 16/05/15 Суб 16:31:34 #261 №179940 
>>179887
Круто. Но он же улетел. Или обещал вернуться?
Аноним 16/05/15 Суб 17:08:47 #262 №179963 
>>179816
а вы не думали что инопланетяне отказались от радиоволн- все эти вай фаи, СВЧ, длинные волны это устаревшее говно для них. Они юзают нейтринную связь пока Земляшки понтуются глонассом, джи пи эсом, вай фаями , цифровым ТВ , и прочей морально устаревшей хуйней.

https://ru.wikipedia.org/wiki/%CD%E5%E9%F2%F0%E8%ED%EE
Аноним 16/05/15 Суб 17:10:37 #263 №179964 
>>179963
Пруфов конечно не будет.
Тебе в сайфач
Аноним 16/05/15 Суб 19:06:30 #264 №180036 
14317923909210.jpg
>>176350
Вот значит есть МКС и она обитаемая, и обитают там преимущественно мужики и обитают обычно долго. Так вот вопрос: как там дело обстоит с дрочкой? человек по сути своей хочет ебаться или в край быть ебаным инстинкт размножение как никак, недалеко мы еще ушли от животных тоесть за дохуя месяцев времяприпровождения надо сбрасывать свой тестестерон чтобы не уничтожать все нахуй и не кидаться на тех редких тней которые на МКС все же попадают. Может там все продумано и дают в снабжении спец фап носки? или все происходит тайком ?
Аноним 16/05/15 Суб 19:09:49 #265 №180037 
>>180036
> человек по сути своей хочет ебаться или в край быть ебаным инстинкт размножение как никак, недалеко мы еще ушли от животных
Говори за себя, плебс.
Аноним 16/05/15 Суб 19:20:53 #266 №180039 
>>180037
я и говорю за себя, я человек здоровый физически и вродде как психически, и хочу ебаться, как большинство твоих мам пап и дидов, не то что ты
Аноним 16/05/15 Суб 19:21:18 #267 №180040 
>>180037
При чем тут. Это чисто физиологическая потребность. Так же как если не пожрать то уныло себя чувствовать будешь.
Аноним 16/05/15 Суб 19:39:23 #268 №180042 
>>180040
>физиологическая потребность
>уныло себя чувствовать будешь
Длительное отсутствие ебли можно пережить, в отличие от отсутствия еды.

>>180039
По этому ты и плебс, не могущий в усмирение своей плоти.
Аноним 16/05/15 Суб 19:46:29 #269 №180045 
>>180042
ошибаешься. длительное отсутствие ебли ну или дрочки неплохо изменят твой гормональный фон. а из этого могут вылезти проблемы в твоем поведении и перемены приоритетов
Аноним 16/05/15 Суб 20:18:37 #270 №180064 
>>180042
Двачую патриция. Неспособность представить себе увлекательную деятельность, полностью занимающую разум - удел тупых плебеев. Их же удел и разговоры типа "низзя без дрочки, гы-гы, нада справлять есесвенные нужды, гы-гы, палюбасу в касмасе там касманавты передергивают, гы-гы". Пиздец, как дети малые. Может и передергивают, но, как и на Земле, это остается личным делом каждого. Приспособление, ну вообще охуеть. В салфетку настрелял, да в мусор. Можно и в подгузник, потом водичкой разбавить. Уж подгузник на содержимое никто проверять не будет. Только следить надо, чтобы какая-нибудь ретивая капля не полетела в другой конец модуля, а то если кому-нибудь в ноздрю залетит, то не хорошо будет.
Аноним 16/05/15 Суб 20:28:42 #271 №180065 
>>180045
Гормональный фон вызывает проблемы с поведением у истеричек разного пошиба, а нормальный уравновешенный человек оправданные пол-года воздержания спокойно переживет. Привыкли сосачеры все по себе мерить. Конечно, бесконечное монотонное сидение за пекой в закрытой комнате приучит по два-три раза в день тилибонькать. Однако если работать в коллективе и заниматься чем-то увлекательным, то рука к письке гораздо реже будет тянуться. Ну и примеров пожизненного воздержания предостаточно, и ничего, живут люди, деревья ебать не торопятся.
Аноним 16/05/15 Суб 20:50:42 #272 №180077 
>>180067
Это все индивидуально. У кого-то спермабак при воздержания раз в месяц-два течет. Ну а если уже недельный нофапон приводит к безостановочной "течке", то проще наверное спускать, да.
Аноним 16/05/15 Суб 22:53:11 #273 №180116 
Сосоны, кефиробляди, это те, кто верят в существование эфира, да?
Аноним 16/05/15 Суб 23:16:03 #274 №180122 
>>180116
Нет, те - кто отрицает его существование.
А верят кефиробоги
Аноним 16/05/15 Суб 23:20:13 #275 №180124 
>>179887
в чем это сделано, обсчитано ? чето гугл копротивляется не хочет искать
Аноним 16/05/15 Суб 23:38:29 #276 №180133 
>>180124
На серьезный тулкит вроде GMAT непохоже, скорее всего какой-нибудь простой симулятор гравитации.
Аноним 16/05/15 Суб 23:39:49 #277 №180136 
>>180116
Нет, верят в отсутствие эфира релятипидоры. Эфиробоги же исследуют реальность.
Аноним 17/05/15 Вск 00:04:14 #278 №180153 
>>180133
ок gmat нашел, но чую будет сложновато, есть что простое, задал параметры орбиты и смотришь как оно развивается
Аноним 17/05/15 Вск 00:30:05 #279 №180167 
>>180116
>Сосоны, кефиробляди, это те, кто верят в существование эфира, да?
Это те, кто любят уринотеррапию в спейсаче. Жить без нее не могут. Кефир - это так, прикрытие, что бы их почаще обливали.
Аноним 17/05/15 Вск 01:20:48 #280 №180183 
14318148487340.jpg
>>180136
Ага, посредством этого самого эфира употребления.
Да и хуле в него верить, вот пруфпик.
Аноним 17/05/15 Вск 04:17:40 #281 №180202 
14318254606750.jpg
Смотрю Lexx, и тут в третьем сезоне увидел интересную штуку - вращающиеся вокруг друг друга планеты с общей атмосферой. И они там летали между планетами просто поднимаясь вверх до зоны влияния другой планеты. Возможна ли такая система?
Аноним 17/05/15 Вск 05:12:05 #282 №180208 
>>180202
чем ближе тела и чем они массивней тем больше эффект приливных сил
представь стоит такой эверест и весит 0.1 своей массы а через 12 часов он уже весит 1.3 (к примеру) хм я бы на это посмотрел, мне кажется что оно бы быстро кору продолбило
Аноним 17/05/15 Вск 05:20:47 #283 №180212 
А если планеты будут обращаться вокруг общего центра тяжести? Т.е. всегда будут повёрнуты друг другу одни стороной. Это не минимизирует торможение об атмосферу?
Аноним 17/05/15 Вск 05:38:49 #284 №180213 
14318303297090.jpg
>>180212
> всегда будут повёрнуты друг другу одни стороной
Это называется приливный захват.

>>180202
Технически да, если атмосфера обеих планет простирается дальше предела Роша, и они находятся в приливном захвате друг у друга. Но если так будет, то
1. Будут пиздецовые вихри и возможно постоянный ветер в одну сторону вокруг планеты с мясосрывающей скоростью.
2. Если газа хотя бы под минимальным давлением хватит аж до местной точки либрации между двумя планетами, и планеты будут размером с Землю, то на поверхности будет давление как на ёбаной Венере. Можно будет летать, размахивая руками. (целых полсекунды, пока тебя не раздавит к хуям). Если предположить, что они летают через вакуум, разогнавшись на атмосферном движке, то проблема становится проще, но не исчезает.

В общем со сколько-нибудь организованной жизнью данная хуета вряд ли будет совместима.
Аноним 17/05/15 Вск 05:47:15 #285 №180214 
>>180213
Жаль, такая-то годнота. Ну а может быть чтобы две планеты аналогичные земле вращались на относительно небольшим расстоянии, чтобы можно было на ракете типа существующий прямой наводкой пулять.
Аноним 17/05/15 Вск 05:49:48 #286 №180215 
>>180214
Такое в принципе возможно, да. Плутон-Харон, только побольше.
Аноним 17/05/15 Вск 08:08:22 #287 №180220 
>>180213
>если атмосфера обеих планет простирается дальше предела Роша
Вангую, что при таком раскладе случится то, что бывает в тесных двойных звездных системах и если одна из планет заполнит своей атмосферой предел Роша, то вторая планета начнет ее пиздинг, в промышленных объемах.
Аноним 17/05/15 Вск 09:21:25 #288 №180225 
>>180136
>релятипидоры
Нахуя так громоздко? Есть же оухенная погремуха "релявы". Столько негативных ассоциаций возникает и при этом такое звучное. Я когда впервые прочитал, то мне так понравилось, что хотел даже кефироебланом притворится и обозвать кого-нибудь так.
Аноним 17/05/15 Вск 11:58:14 #289 №180234 
>>179628
>это видео
Вычеркнул гравитацию из очереди на просмотр. Уж на что я киноговноед, но всему есть предел.
Аноним 17/05/15 Вск 12:06:33 #290 №180237 
>>180220
Предполагается что они одинаковые и атмосфера похожая - состав давным-давно уравнялся.
Аноним 17/05/15 Вск 12:12:00 #291 №180238 
>>180234
Я знал на что шёл и смотрел ради потрясающей картинки. А ещё я дрочу на МКС, да
Аноним 17/05/15 Вск 12:19:27 #292 №180242 
>>180036
Читал интервью с Виноградовым, он говорил, что обычно устаёшь так, что на дрочку нет ни сил ни охоты.
Аноним 17/05/15 Вск 16:00:40 #293 №180306 
Есть вопрос. Суть такова - возможно ли создать искусственную гравитацию на МКС (или что там сейчас вместо нее планируется), путем вращения (да, как в интрестелларе). Если можно то какие подводные камни, и насколько дорого с точки зрения топлива и т.п. будет это стоить.
Аноним 17/05/15 Вск 16:06:38 #294 №180308 
>>180306
Технически очень сложно.
Аноним 17/05/15 Вск 16:09:06 #295 №180310 
>>180308
В чем сложность то? Доставлять станцию можно отсеками, собирать уже на орбите, будет такой многоугольник из 5 отсеков, добавить пару двигателей придающих ускорение, PROFIT.
Аноним 17/05/15 Вск 16:09:27 #296 №180311 
>>180306
Здесь все разжевано
http://universeru.com/2013/05/problema-nevesomosti-iskusstvennaya-gravitaciya-za-schet-vrashheniya/
Аноним 17/05/15 Вск 16:10:05 #297 №180312 
>>180310
Доставлять нечем уже. Шаттл - все.
Аноним 17/05/15 Вск 16:23:31 #298 №180315 
>>180312
Я думаю что именно эта проблема преувеличена, запилить грузовик для доставки-стыковки пассивных модулей можно, и он будет проще шаттла. Шаттл просто уже был на тот момент, когда строили МКС.

>>180310
>В чем сложность то?
В нагрузках на конструкцию и надежности, в размерах, да и просто в теормехе. Чтобы избежать вредного градиента ускорения, нутации, биений - колесо должно быть ебическим, надо равномерно распределять массу, а лучше обладать активной стабилизацией при помощи ёба-гироскопа (ходящие космонавтики пиздец разболтают его, если этого не будет). Колесо надо закручивать-раскручивать, чтобы делать маневры, ибо оно самостабилизируется и противостоит попыткам его переориентировать. Алсо, сложность именно в сложности. Даже лоскутная МКС это система ебической сложности. В целом на современном технологическом уровне станцию-колесо запилить малореально. Тросовую систему с двумя кабинками и то проще сделать, чем колесо. вот только MMOD ебанёт по тросу, и всё улетит на Луну Это все технические моменты, понятно что финансово никто этим заниматься не будет, да и смысла в этом особого нет.
Аноним 17/05/15 Вск 17:38:33 #299 №180335 
14318735136300.jpg
Сап спесач разреши мой спор,поясни за засирание орбиты.Допустим если вывести капсулу полную стальных болтов и взорвать на орбите,останутся ли они там вращаться на дикой скорости круша спутники и МКС
Аноним 17/05/15 Вск 18:12:59 #300 №180347 
>>180335
Нет. Надо основательно засрать орбиту именно на высоте МКС и на пересекающихся курсах. Но понадобится столько металлолома, что дешевле ракетой ебнуть.
Аноним 17/05/15 Вск 18:19:50 #301 №180348 
>>180335
Ведро с болтами это платина спейсача уже.

Если взорвать равномерно в разные стороны- 3/4 попадают обратно на следующем витке, потому что сменят орбиту на цепляющую атмосферу. Останутся только те что разлетятся в стороны и сменят наклонение. Часть из них столкнется друг с другом через виток, т.к. прилетят в ту же точку (но не совсем, из-за возмущений). С низкой орбиты попадают (мелкие частицы сходят крайне быстро), на высокой плотность засирания будет очень невелика, см http://en.wikipedia.org/wiki/Big_sky_theory (тем более орбиты выйдут эллиптические из-за стартового импульса).
Аноним 17/05/15 Вск 18:59:02 #302 №180356 
>>180335
Скорость на низкой околоземной орбите около 7,7 км\с. Скорость разлета осколков например гранаты Ф-1 - 0,7 км\с. Т.е. осколки образуют достаточно небольшой по ширине и высоте коридор, не считая тех, что черканут атмосферу и сгорят.
Аноним 17/05/15 Вск 23:28:41 #303 №180467 
>>180315
> Тросовую систему с двумя кабинками и то проще сделать, чем колесо
Я попробовал в огурцаче. Осевую бочку шатает как ни крути, а если она тяжёлая, то нагрузки на тросы неебические, рывками. Стыковаться к оси малореально. Ну, в кербале всё можно, а ирл я бы зассал. И раскрутка тросовой конструкции - тот ещё аттракцион, сомнительный весьма.
Аноним 17/05/15 Вск 23:43:07 #304 №180481 
14318953874240.jpg
Анон, я не понимаю, как сверхсветовое движение может нарушить принцип причинности. Вот, допустим, я прошёл сквозь червоточину на несколько тысяч световых лет и тут же нырнул обратно. Везде написано, что я увижу себя, проходящего сквозь червоточину. Но ведь я уже прошёл сквозь неё, блять! Как тогда я увижу себя?
Аноним 18/05/15 Пнд 00:45:28 #305 №180500 
>>180481
Дело в том, что из теории относительности получается, что тело движущееся со сверхсветовой скорость, также движется назад во времени.
Аноним 18/05/15 Пнд 00:51:53 #306 №180505 
>>180500
Срань господня.
Аноним 18/05/15 Пнд 01:10:03 #307 №180511 
>>180500
Но это же круть несусветная
Аноним 18/05/15 Пнд 01:57:21 #308 №180531 
Я верю в эфир, пацаны.
Аноним 18/05/15 Пнд 01:57:57 #309 №180533 
Я верю в эфир, пацаны.
Аноним 18/05/15 Пнд 02:03:00 #310 №180540 
>>180531
Где достать? Я бы тогда тоже верил.
Аноним 18/05/15 Пнд 03:55:03 #311 №180558 
>>180531
Это такой тонкий реквест этой пасты?
У нас было 2 пакета торсионных полей, 75 инерциоидов, 5 упаковок креационистских теорий, пол-солонки волнового генома Гаряева и целое множество гомеопатии всех сортов и расцветок, а также френология, телегония, ящик мичуринской биологии ,парапсихология , новая хронология Фоменко и эфир. Не то что бы это был необходимый запас для поездки. Но если начал собирать дурь, становится трудно остановиться. Единственное что вызывало у меня опасение - это эфир. Нет ничего более беспомощного, безответственного и испорченного, чем эфирные зомби. Я знал, что рано или поздно мы перейдем и на эту дрянь.
Аноним 18/05/15 Пнд 04:07:01 #312 №180561 
>>180558
Или этой?
Чёртов эфир... после него вас развозит так, что вы похожи на пьяницу из российской академии естественных наук: полная утраты математических навыков, галлюцинации и искаженное восприятие физических законов, мозг патологически ищет гранты на гениальные изобретения, начинается юдофобия, мобильник с GPS начинает нести чушь вместо координат. Интересное состояние когда все видишь, но не в силах что либо контролировать. Подходишь к трибуне чётко зная, что надо признать свои ошибки, чтобы уйти без позора. Но на месте всё происходит не так. Тебя отпихивает злобный ученый, а ты думаешь: в чём дело, что происходит? ...И слышишь собственный лепет: Ацюковского трахнула собака, так и ему и надо.
Аноним 18/05/15 Пнд 09:32:19 #313 №180583 
Допустим, я лечу по орбите. У меня есть пистолет.
Я могу выстрелить в 6 направлениях. Что произойдет в каждом из этих случаях? Стрелять я буду на уровне центра тяжести, вращения не будет происходить.
Аноним 18/05/15 Пнд 10:34:01 #314 №180588 
>>180583
Разъяснение в огурцевики с картинками (орбита против часовой стрелки) http://wiki.kerbalspaceprogram.com/wiki/Maneuver_node/ru#.D0.AD.D1.84.D1.84.D0.B5.D0.BA.D1.82.D1.8B_.D0.BD.D0.B0_.D0.BE.D1.80.D0.B1.D0.B8.D1.82.D0.B5
Естественно, эффект от пистолета будет почти незаметен.
Аноним 18/05/15 Пнд 10:51:17 #315 №180590 
>>180588
Я лечу в скафандре.
Аноним 18/05/15 Пнд 11:04:22 #316 №180592 
>>180590
Тогда эффект будет более заметен но принцип взаимодействия не изменится (тут пистолет будет играть роль импровизированного ТТРД со всеми вытекающими). Кстати при выстреле по нормали к орбите есть шанс через один виток снова повстречать пулю
Аноним 18/05/15 Пнд 11:31:22 #317 №180599 
У самолетов есть магниевые части, можно их поджечь спичкой?
Аноним 18/05/15 Пнд 11:33:00 #318 №180600 
>>180599
Там магниевые сплавы. Их нельзя поджечь спичкой.
Аноним 18/05/15 Пнд 11:36:07 #319 №180601 
В чем отличие Протона от Союза, почему используют разные ракеты для одних целей?
Аноним 18/05/15 Пнд 11:37:57 #320 №180602 
>>180601
>Протон и Союз
>для одних целей
Уважаемый, вам надо продышаться на свежем воздухе.
Аноним 18/05/15 Пнд 11:40:18 #321 №180603 
>>180602
Он эфира надышался. Сейчас отпустит.
Аноним 18/05/15 Пнд 12:37:24 #322 №180605 
>>180603
Хорошо хоть не гептила.
Аноним 18/05/15 Пнд 12:46:24 #323 №180608 
А Союз может в ГСО?
Аноним 18/05/15 Пнд 12:53:23 #324 №180609 
>>180608
СоюзФГ с РБ "Фрегат" Венеру-Экспрес закинули аж на орбиту Венеры.
Аноним 18/05/15 Пнд 19:36:40 #325 №180672 
>>180609
>на орбиту Венеры
Строго говоря, РН с РБ подкинули до Венеры, а там уже сама АМС тормознула.
Аноним 18/05/15 Пнд 20:16:22 #326 №180677 
14319693821290.jpg
Чубака, че за полосы?
Аноним 18/05/15 Пнд 20:16:46 #327 №180678 
14319694062200.jpg
>>180677
Аноним 18/05/15 Пнд 20:23:11 #328 №180679 
14319697910480.jpg
>>180677
RAWRGWAWGGR GGGWARRRHHWWWW GGWWWRGHH WAGRRRRWWGAHHHHWWWRRGGAWWWWWWRR.
Аноним 19/05/15 Втр 01:09:52 #329 №180765 
Часто на видео запусков вижу некую ебалу, вентилирующую (?) пн под обтекателем, причём это касается как спутников, так и пилотируемых кораблей.
Что это и нахуя?
Аноним 19/05/15 Втр 01:10:53 #330 №180766 
>>180765
Сделай скрин.
Аноним 19/05/15 Втр 01:18:55 #331 №180768 
>>180766
All of the electrical connections were made and the fairing was ventilated with clean, cooled air to keep the internal humidity at bay and both satellites at a controlled temperature.

Неужели это настолько важно, что на поезде, тягающем ракеты на Байконуре, есть специальный вагон-кондиционер? Поясните, зачем всё так сложно, КА же не страшны перепады температуры и даже влажность в разумных пределах, нет?

Аноним 19/05/15 Втр 01:21:37 #332 №180769 
>>180768
Электрика боится влаги. На земле влага есть, в космосе практически нет. Я думаю так
Аноним 19/05/15 Втр 01:48:38 #333 №180783 
>>180768
Чем меньше нерасчетных воздействий между сданными испытаниями и эксплуатацией, тем меньше вероятность проебаться.
Аноним 19/05/15 Втр 02:29:24 #334 №180790 
>>180768
Конденсат может помешать. Он выпадает росой, может быстро замерзнуть, при пуске, приморозить, например что-нибудь к чему-нибудь и так далее.
Аноним 19/05/15 Втр 05:54:00 #335 №180810 
>>180768
> Неужели это настолько важно, что на поезде, тягающем ракеты на Байконуре, есть специальный вагон-кондиционер?
Да, это очень важно. КА необходимо термостатировать. Допустимые перепады в бортовом отсеке обычно 20+-5 градусов. Это необходимо обеспечивать при любой внешней погоде - отсюда и такая ебола.
Аноним 19/05/15 Втр 12:11:48 #336 №180824 
Как МКС ориентирована относительно земной поверхности? Вращается ли она вокруг своей оси с периодичностью равному одному обороту вокруг планеты, подобно тому как это делает луна? Если да, то каким образом регулируется эта скорость вращения, автоматически, или же при коррекции всей орбиты? МКС это же огромная станция с оче большой парусностью. Как к ней стыкуются союзы если она вращается вокруг своей оси? Как в интерстоляре?
Аноним 19/05/15 Втр 12:13:01 #337 №180826 
>>180824
*периодичностью равной
Аноним 19/05/15 Втр 14:41:28 #338 №180850 
14320356885390.jpg
14320356885391.jpg
>>180824
Там есть несколько разных режимов ориентации - XVV, YVV, XPH, XPOP, ZLV, ZVV и т.п. Некоторые из них с постоянной ориентацией на Землю, некоторые с постоянной ориентацией на Солнце, некоторые удобны для стыковки Союзов или Шаттлов (тем нужно было минимизировать риск повреждения теплозащиты микрометеоритами, поэтому повернуты должны были быть одной стороной), некоторые использовались только на ранних этапах строительства.

Изначально выбор каждого зависел от бета-угла (угол между орбитой МКС и направлением на Солнце), т.к. он определяет, как часто МКС будет затеняться Землей, и соответственно температурный режим и инсоляцию станции. Сейчас станция летает только в режиме XVV, одной стороной всегда к Земле, снижая риск ударов MMOD и снижая драг. При малом бета-угле солнечные панели еще и складываются в тени Земли, уменьшая сечение и парусность.
Аноним 19/05/15 Втр 14:44:01 #339 №180851 
>>180850
Только не складываются, а поворачиваются.
Аноним 19/05/15 Втр 15:50:07 #340 №180863 
14320398072570.jpg
14320398072671.jpg
А чем на МКС занимаются? Там же делать по сути нечего. Ну, может, биоэксперименты с половой еблей... Кстати, если кончить в невесомости тян на животик, сперма к нему приклеится или соберётся в шарики и будет летать по всей МКС?
Аноним 19/05/15 Втр 17:07:24 #341 №180887 
>>180863
>А чем на МКС занимаются?
ВНЕЗАПНО, наукой.
https://en.wikipedia.org/wiki/Scientific_research_on_the_International_Space_Station
В основном-таки биологией и физикой.

>Кстати, если кончить в невесомости тян на животик, сперма к нему приклеится или соберётся в шарики и будет летать по всей МКС?
Если попадёшь - приклеится, если промахнёшься - полетит прямо до первого препятствия и приклеится уже к нему.
5
Аноним 19/05/15 Втр 17:14:30 #342 №180889 
>>180863
По моему чтобы кончить необходима гравитация. Без нее она просто не выльется из хуя.
Вот тоже подтолкнул к вопросу. Как ссут в космосе? Как бы чтобы выссать моча должна скопиться в нижней части мочевого пузыря. А так в космосе же она пузырями. В топливных баках эту проблему решают ускорением или там нагнетают с помощью гащза.
Аноним 19/05/15 Втр 17:27:11 #343 №180899 
>>180889
Попробуй поссать, стоя на голове.
Аноним 19/05/15 Втр 17:28:52 #344 №180900 
>>180899
Двачую, охуенные ощущения, ещё захочешь. Будто в мочевом пузыре что-то нежно перемещается, доставляя чуть ли не оргазмы.
Аноним 19/05/15 Втр 17:42:40 #345 №180907 
>>180889
>По моему чтобы кончить необходима гравитация.
Ноуп. Мышечные сокращения же.
Аноним 19/05/15 Втр 17:54:01 #346 №180909 
>>180889
>Как ссут в космосе
Насадка с небольшим понижением давления, с отсосом, так сказать нет?
Аноним 19/05/15 Втр 18:04:26 #347 №180912 
>>180909
Спейсач отупел.
Эта хуйня нужна только для того, чтобы моча не разлеталась по кораблю.
Мочевой пузырь - очень эластичная штука, и "наддувать" его не нужно, очевидно.
Блджад, надуй шарик, и подумай, почему он сдувается, если открыть дырку.
Сперма же вылетает под действием мышц.
Аноним 19/05/15 Втр 19:40:25 #348 №180934 
>>180863
> А чем на МКС занимаются?
Перед очередным полетом в журнале "Новости космонавтики" публикуется программа экспериментов для экспедиции. Там дохуя всего. Удивительно, что они вообще все успевают. Ради интереса посмотри подборку за какой-нибудь год.
Аноним 19/05/15 Втр 22:32:29 #349 №181000 
>>176350
В чём профит поиска внеземной жизни, кроме очевидного "утереть нос верунам?"
Аноним 19/05/15 Втр 22:38:23 #350 №181004 
>>181000
А зачем вообще учиться (заниматься наукой)?
Аноним 19/05/15 Втр 22:42:14 #351 №181006 
Почему "Магеллан" завёрнут в белую теплоизоляцию, а "Галилео" почти весь - чОрный?
Аноним 19/05/15 Втр 22:46:58 #352 №181010 
>>181004
Чтобы приносить пользу человечеству, очевидно.
Аноним 19/05/15 Втр 23:07:33 #353 №181023 
>>181006
Очевидно потому что Галилео летит труд где осне мало солнечной энергии а магеллан - туда где ее осне много. Обьяснять надо?
Аноним 19/05/15 Втр 23:08:55 #354 №181024 
>>181000
Когда обезьяна осознала, что ее интересуют не только вопросы "где переночевать, попить, пожрать и с кем поебаться", то начала спрашивать всякую хуиту, типа "почему небо голубое, трава зеленая и откуда все взялось?", немного погодя возник еще вопрос "Мыче тут одни штоле блядь, ау, епта!?"
Короче это фундаментальный вопрос, мучающий людей с древних времен.

>>181010
>Чтобы приносить пользу человечеству, очевидно.
Нет никакой практической пользы человечеству, от обнаружения странных вихрей на Сатурне, или обнаружения округлых камней на Титане. Профит есть от решения задач, при постройке всякой ебалы, которая полетит фотографировать Сатурн и садиться на Титан.
Аноним 19/05/15 Втр 23:27:28 #355 №181032 
>>181024
Возможно от вихрей на Сатурне пока нет пользы. В своё время потешались над ебанутыми которые меряли длину тени от палки в разные дни. А смотри ты, и профит получился.
Аноним 19/05/15 Втр 23:56:02 #356 №181054 
>>181032
Возможно от поиска внеземной жизни пока нет пользы. В своё время потешались над ебанутыми которые меряли длину тени от палки в разные дни. А смотри ты, и профит получился.
Аноним 20/05/15 Срд 00:02:05 #357 №181060 
>>181054
Ну я у тому что неизвестно какой побочный профит из этого получится. Полёты в космос начало тоже было сорт оф меряние пиструнами, а в итоге имеем ЖПС, гелогоразведку, спутниковую связь, и прочие ништяки.
Аноним 20/05/15 Срд 01:33:56 #358 №181127 
>>181060
Я не оспариваю профита от изучения космоса. Что ты ко мне пристал?
Аноним 20/05/15 Срд 02:04:00 #359 №181150 
>>180810
Почему 20? А потом сразу открывается в космосе и всё равно там будет перепад. Может важнее абсолютную сухость воздуха там поддерживать?
Аноним 20/05/15 Срд 02:20:13 #360 №181157 
>>181127
Выебать тебя хочу
Аноним 20/05/15 Срд 13:05:12 #361 №181228 
>>176350
Почему космическая гонка ВСЕ? И возможно ли её повторение в 2020-х-2050-х годах с участием частников?
Аноним 20/05/15 Срд 13:13:43 #362 №181230 
>>181228
>Почему космическая гонка ВСЕ?
Потому что СССР - все. Не с кем фаллометрию устраивать.
>И возможно ли её повторение в 2020-х-2050-х годах с участием частников?
Нет, частники свои деньги считать привыкли. Только если на Марсе обнаружат какой-нибудь несимметричный ЕбанаторийПиздория, ложка которого будет способна обеспечить небольшой город электричеством на год.
Аноним 20/05/15 Срд 13:19:34 #363 №181231 
>>181230
How about Гелий-3 на Луне?. Вполне себе "ЕбанаторийПиздория, ложка которого будет способна обеспечить небольшой город электричеством на год. "
Аноним 20/05/15 Срд 13:28:49 #364 №181234 
>>181231
>How about Гелий-3 на Луне?.
Хау эбаут достроить ИТЭР и научится в термоядерную энергетику для начала, успев до 2050-го?
Алсо концентрация Гелия3 на Луне такая, что надо перемолоть 100500 тонн, что бы намолоть хотя бы наперсточек. Придумали хуйню и форсят всюду.
Аноним 20/05/15 Срд 13:29:41 #365 №181235 
>>181234
>научиться
fix
Аноним 20/05/15 Срд 14:48:04 #366 №181252 
>>181234
1 грамм на сто тонн. Один грамм по энергетике как 15 тонн нефти. Выгодно же.
Аноним 20/05/15 Срд 15:07:43 #367 №181257 
>>176565
А как ты его наденешь?
Аноним 20/05/15 Срд 15:08:00 #368 №181258 
>>176565
А как ты его наденешь?
Аноним 20/05/15 Срд 15:10:26 #369 №181259 
>>181258
Че сложного-то? Взял и одел.
Аноним 20/05/15 Срд 15:26:26 #370 №181261 
14321247866420.gif
Почему космос черный? Я знаю ответ, просто траллирую
Аноним 20/05/15 Срд 15:40:59 #371 №181265 
>>181261
Другой краски не было
Аноним 20/05/15 Срд 16:19:40 #372 №181266 
>>181261
Потому что фотометрический парадокс.
Аноним 20/05/15 Срд 16:48:36 #373 №181273 
>>181266
Развернутый ответ зашел бы лучше
Аноним 20/05/15 Срд 18:09:42 #374 №181292 
>>181261
Потому что фотоны теряют энергию в эфир и смещаются в сторону радиодиапазона. Поэтому далекие объекты светят тусклее и краснее. На совсем далеких объектах имеется максимум светимости соответствующий реликтовому излучению. Еще дальше объекты практически не видны ибо фотоны растеряли всю энергию.
https://2ch.hk/sci/res/238438.html#245051
Аноним 20/05/15 Срд 18:55:35 #375 №181301 
Существуют ли снимки движений звезд в плотных шаровых скоплениях?
Аноним 20/05/15 Срд 19:07:44 #376 №181303 
1)Правильно ли я расположил затраты для одинакового груза:
- Вывести на низкую опорную орбиту
- Вывести груз на ВЭО
- Вывести груз на ГСО
- Облететь грузом вокруг Луны
- Выйти на орбиту Луны
- Улететь на Марс
2) Какой (какие) космонавты были дальше всех от Земли, если исключить миссии, связанные с луной? На ГСО или ВЭО люди были?
Аноним 20/05/15 Срд 19:56:16 #377 №181309 
>>181303
Неправильно.
1. Высокоэллиптической орбитой можно назвать все что угодно, апогей может быть хоть на границе сферы Хилла Земли.
2. Существует экзотический маневр выхода на ГСО с облетом Луны (!), который дешевле чем прямой вылет на ГСО при пуске с высокоширотных космодромов (Байконур например). Правда ИРЛ он никогда не применялся, потому что до Луны туда-обратно 8 дней лететь, РБ столько не протянут, да и нагрузка простаивает почём зря.

Дальше низкой орбиты не летали.
Аноним 20/05/15 Срд 19:58:33 #378 №181310 
Опять кефироблядский шабаш в треде. Философский камень лучше ищите идите.
Аноним 20/05/15 Срд 20:10:15 #379 №181315 
>>181252
>Один грамм по энергетике как 15 тонн нефти. Выгодно же.
Че не очень, сколько из 15 тонн получится керосину? А сколько керосина надо туда и обратно? То-то же.
Аноним 20/05/15 Срд 20:14:07 #380 №181317 
>>181292
Кефироблядь приплыла, принесла симуляцию статической Вселенной. Еще больше наркомании не было?
Аноним 20/05/15 Срд 20:14:57 #381 №181318 
>>181301
Нет, сколько не искал - снимков не нашел. Даже симуляций чет нету, или я хуево искал.
Аноним 20/05/15 Срд 20:27:40 #382 №181325 
14321428611110.gif
>>181301
Вот нашел какую-то занюханную симуляцию.
Аноним 20/05/15 Срд 22:34:01 #383 №181345 
14321504417820.jpg
14321504417931.gif
>>181301
http://www.youtube.com/watch?v=klDDU0ijXyE
Аноним 20/05/15 Срд 22:37:08 #384 №181348 
>>181303
2)Gemini 11 crew Charles Conrad, Jr and Richard F. Gordon, Jr. fired their Agena Target Vehicle rocket engine on 14 September 1966, at 40 hours 30 minutes after liftoff and achieved a record apogee altitude of 1,374.1 km (853.8 mi).
Аноним 20/05/15 Срд 22:38:31 #385 №181349 
Поясните мне про "Орион", тот который на третьем пике тут >>181166
Почему у него импульс выше,чем у менее ёбистых систем? Ведь каждый заряд - это сложное устройство, да и ресурс щита тоже не бесконечен, т.е. дохуя всякого лишнего материала приходится везти и расходовать.
Аноним 20/05/15 Срд 22:47:15 #386 №181354 
>>181349
>Ведь каждый заряд - это сложное устройство
Это не помешало наклепать тысячи этих зарядов, а тактического ЯО - вообще никто не считал никогда. И сколько этих спецбоеприпасов - вряд ли кто знает. Полагаю, что десятки тысяч, с каждой стороны.
>да и ресурс щита тоже не бесконечен
Не бесконечен, конечно, но плиту на жопе можно вполне крепкую сделать.
Аноним 20/05/15 Срд 22:59:52 #387 №181365 
>>181345
Они так медленно движутся, что за их движениями практически не уследить. Я думал движения будет заметнее. А вон как оказывается, нужны десятки тысяч лет чтоб такое увидеть.
Аноним 20/05/15 Срд 23:42:57 #388 №181388 
>>181365
Ну конечно, расстояния же пиздецовые.
Аноним 21/05/15 Чтв 04:04:39 #389 №181450 
>>181234
>Алсо концентрация Гелия3 на Луне такая, что надо перемолоть 100500 тонн, что бы намолоть хотя бы наперсточек. Придумали хуйню и форсят всюду.

зачем молоть - видел такие штуки разогревающие асфальт чтобы заплатки ставить, тебе поверхностный слой подогреть само вылезет

>>181252
хз как с нефтью, а с условным топливом в 29 мегаджоулей за кг - один грамм смеси дейтерий + гелий3 это эквивалент 9100 кг этого самого условного топлива
Аноним 21/05/15 Чтв 04:32:55 #390 №181451 
>>181450
Посчитай площадь, которую придется обработать, что бы собрать килограммчик Гелия3.
>само вылезет
Слишком влажно у тебя получается. Харвестр, у которого все само.
Аноним 21/05/15 Чтв 10:40:57 #391 №181461 
>>176350
В случае глобальной ядерной войны были ли видны из космоса вспышки взрывов как это часто любят показывать?
Аноним 21/05/15 Чтв 10:56:32 #392 №181463 
>>181315
>А сколько керосина надо туда и обратно?
Ключевые технологии имеют, по оценке НАСА, уровень технологической готовности 7/10. Рассматривается возможность производства большого объёма электроэнергии, равного 1 ПВт. При этом стоимость лунного комплекса оценивается примерно в 200 трлн долл. США. В то же время стоимость производства сравнимого объёма электроэнергии наземными солнечными станциями — 8000 трлн долл. США, наземными термоядерными реакторами — 3300 трлн долл. США, наземными угольными станциями — 1500 трлн долл. США

Ах да Ещё на Луне есть йоба вакуум что годно для металургии и изготовления электроники
Аноним 21/05/15 Чтв 11:25:34 #393 №181467 
>>181463
>200 трлн долл. США
3-х летний общемировой ВВП? Откуда инфа? Такая сума эквивалентна строительству сотни городов миллиоников с нуля в Сибири. Я прекрасно осознаю сложность возведения целого промышленного космоса на Луне, но число 200 трлн. все равно подвергаю сомнению.
Аноним 21/05/15 Чтв 11:33:00 #394 №181468 
>>181463
Олсо что это за объем электроэнергии, измеряемый в ваттах? В ваттах измеряется мощность. Привычная нам единица объема электроэнергии - киловаттчас, и если перевести их в петаватты, то этих петоваттовчас на Земле уже производится 23 в год. Ты там числа не на ходу выдумываешь?
Аноним 21/05/15 Чтв 11:37:50 #395 №181469 
>>181461
Зависит от погоды, времени суток, расстояния до планеты и мощности зарядов, я так думаю. Но точно не так, как в кино. Раскаленный шар в пару километров и ядерный гриб высотой 10-15 км и примерно столько же в диаметре, не очень то заметен на фоне шарика такого радиуса.
Аноним 21/05/15 Чтв 11:46:13 #396 №181470 
>>181463
>Ключевые технологии имеют, по оценке НАСА, уровень технологической готовности 7/10.
Уровень технологической готовности ровно 0, пока обезьяны не смогут в термояд, с положительным выхлопом. Как бы там чего не прикидывало НАСА.
>Рассматривается возможность производства большого объёма электроэнергии, равного 1 ПВт.
Туда же. Они там из чего считают? Из несуществующей установки, с непонятным КПД?
>наземными термоядерными реакторами
НАСА хочет сказать, что выхлоп от наземной станции на том же термояде дороже, чем на Лунной?
Ты откуда это принес, давай ссылку, слишком влажно.
Аноним 21/05/15 Чтв 13:17:25 #397 №181473 
>>181469
А какой силы взрыв должен быть, чтобы как в кино? Царь-бомбы хватит?

разрушители мифов.mkv
Аноним 21/05/15 Чтв 13:35:28 #398 №181474 
>>181473
На ночной стороне планеты достаточно будет обычного ТЯО на тёмном фоне даже достаточно сильные грозы очень хорошо заметны, посмотри таймлапсы с МКС. На дневной-же стороне без оптического оборудования скорее всего будет заметно только что-то уровня "Кузькиной Матери", да. Причём заметен будет не столько сам подрыв сколько его последствия, ибо выжженое и "вылизанное" вызрывной волной пятно размером с Париж незаметить трудно.
Аноним 21/05/15 Чтв 14:13:24 #399 №181476 
>>181474
А излучение не забыли? Не то что заметно будет, оптику повыжигает к чертям
Аноним 21/05/15 Чтв 14:42:30 #400 №181478 
>>181476
>А излучение не забыли?
Забыли. Но разве излучение даже от нескольких десятков детонаций достаточно чтобы
>оптику повыжигает к чертям
у КА?
Аноним 21/05/15 Чтв 14:50:38 #401 №181481 
14322090386170.png
Это что за секретный проект НАСА такой?
Аноним 21/05/15 Чтв 15:12:02 #402 №181482 
>>181478
Ну хуй знает. Матрицу засветит наверно. Я пошутил про выжечь.
Аноним 21/05/15 Чтв 15:33:37 #403 №181485 
>>181482
Не засветит, квадрат расстояния и все такое.
Аноним 21/05/15 Чтв 16:04:54 #404 №181486 
>>181469
С орбиты МКС отлично видно даже салют ночью. Без всяких больших выдержек, как в случае таймлапсов и грозы.
Аноним 21/05/15 Чтв 17:17:07 #405 №181491 
>>181486
>отлично видно даже салют ночью.
Дай посмотреть-то.
>С орбиты МКС
Что-то не помню, что бы вопрошающий что-то говорил про орбиту МКС, равно и про то, что все происходит в ясную погоду ночью.
>больших выдержек, как в случае таймлапсов
Выдержка выставляется с учетом освещенности, и не зависит от того, тайм лапс это, или одиночный кадр. Строго говороя таймлапс - это и есть последовательность одиночных снимков.
Ну и наконец главное: вопрос был, не про видно-не видно, а про то, насколько реалистично это выглядит в фильмах.
Аноним 22/05/15 Птн 00:47:36 #406 №181563 
На каком расстоянии от нас находится галактика NGC 6051?
Аноним 22/05/15 Птн 00:49:56 #407 №181565 
>>181563
Ну хуй его знает
Аноним 22/05/15 Птн 01:07:04 #408 №181570 DELETED
>>181563
Никто не знает
Аноним 22/05/15 Птн 02:27:08 #409 №181593 
>>181565
>>181570
Очень нужно ведь, ананасики.
Аноним 22/05/15 Птн 02:37:10 #410 №181597 DELETED
>>181593
Но это непознаваемо. Возможно эфирная теория при достаточном финансировании могла бы дать ответ.
Аноним 22/05/15 Птн 02:38:24 #411 №181598 
>>181593
Через красное смещение разве нельзя посчитать? - +0,032366 ± 0,000153
я ни фига не понимаю в этом матане но вики говорит вроде можно
Аноним 22/05/15 Птн 03:26:59 #412 №181602 
>>181598
https://youtu.be/1dqLoilEVUk?t=2531
Что-то не сходится. У меня получилось 466 млн св лет, а он говорит о 17.
Аноним 22/05/15 Птн 03:28:37 #413 №181603 
>>181602
На 42:12 это говорит.
https://youtu .be/1dqLoilEVUk?t=2531
Аноним 22/05/15 Птн 03:37:39 #414 №181604 
>>181603
Ты биоэнергохуцов будешь слушать или где?

Заебца что у меня звук не работает
Аноним 22/05/15 Птн 03:38:02 #415 №181605 
*хуйцов
Аноним 22/05/15 Птн 05:58:27 #416 №181610 
>>181563
450-490 млн световых лет.
Аноним 22/05/15 Птн 14:53:22 #417 №181650 
Спейсач, такой вопрос. Находят ли астронавты и астрофизики всякие необъяснимые НЕХ в космосе? Ну чтобы реально: "мы не знаем что эта за хуйня вон там" или "хуй знает что это сейчас было". И как часто?
Аноним 22/05/15 Птн 16:20:54 #418 №181665 
>>181650
https://m.youtube.com/watch?v=lNXcCwLqLhM
Гречко тралил например
Аноним 22/05/15 Птн 16:49:59 #419 №181670 
>>181650
Вся вселенная одна сплошная НЁХ.
Аноним 22/05/15 Птн 16:58:42 #420 №181672 
>>181670
двачую - даже планета земля являеться НЁХ
Аноним 22/05/15 Птн 17:01:03 #421 №181673 
>>181650
Постоянно находят. Навскидку:
Недостающая масса. (т.н. Темная материя холодная и горячая)
Лучи ультра высоких энергий (не ясно, что их рождает)
Гамма-всплески (Не ясен источник)
Темная энергия (не ясно, что заставляет пространство расширяться)
Черные дыры (До сих пор гипотеза, хотя астрофизики анус ставят, что они существуют, но прямых пруфов нет, только косвенные наблюдения эффектов)
Темный поток
Ось зла

Аноним 22/05/15 Птн 21:14:43 #422 №181709 
>>181261
Отвечаю на свой вопрос
https://www.youtube.com/watch?v=rve_x4nBpts
Аноним 22/05/15 Птн 22:34:11 #423 №181727 
Если спрыгнуть с 200 км с парашютом, можно выжить?
Не с МКС, а с неподвижной точки, как Феликс.
Аноним 23/05/15 Суб 00:25:14 #424 №181760 
>>181727
Смотря как и в чем (с чем прыгать) и куда.
Аноним 23/05/15 Суб 01:07:11 #425 №181771 
>>181727
Без теплозащиты нет, нагрев большой будет. Да и перегрузки тоже не сахар.
Аноним 23/05/15 Суб 19:37:39 #426 №181854 
>>181451
давай вводную, плотность слой и тд
насчет само ты даже не потрудился привести пару аргументов, а я уже открыл 20 ресурсов с информацие могущей быть полезной, чтобы хотя бы слегка оценить потенциал

но меня улыбнули эти школьники http://nsportal.ru/ap/library/nauchno-tekhnicheskoe-tvorchestvo/2014/03/18/dobycha-geliya-3-na-lune-kak-alternativa-dobychi
>Как нам стало известно, США уже в следующем году планирует начать строительство станции по добыче гелия-3 на Луне.

так лаптем прикинуть - ок вводная 100 тонн мусора, где 1 грам нужного нам вещества.
просто так нагреть это же не интересно, поэтому по хардкору.
так на вскидку средний атомный вес 24 а.е, возмем щедро 24 э.в. на ионизацию. те в среднем перевод этого риголита в плазму это 100 кДж на грамм, один грамм смеси гелий-дейтерий позволяет нам перевести 3.6т риголита. температура плазмы 288000 кельвинов. температура хлодильника унас ну пусть будет 100-200-300 кельвинов - в данном случае не принципиально. идеальная тепловая машина восстанавливает нам 99.9 процентов, за вычетом всяких восстановительных процессов, но хз считать это мусором или нет, но это вопрос технологии.
ладно по хадкору лаптем не фонтан, думал лучше будетвру конечно, уже считал
нагрев до 3000К, это примерно 0.25 эВ на 1 а.е, те в 100 раз менее затратно, соотвественно 1Г смеси нагревает 360т риголита, он уже во всю плавится, там не только гелий можно выпарить, а с учетом бездонного(буквально Дна Нет) просто холодильника, 80-90 процентный кпд рекуперации вполне реален, это в 5-10 раз больше по итогу. те 1800-3600т риголита. те мы имеем вариант 1Г смеси добывает 3.6Г гелия (те 5Г смеси в итоге). (18-36Г гелия если получше подойти к вопросу). это уже больше 300% прибыли, я уже лечу.
неплохо, осталось эту штуку в которой жечь сделать.
Аноним 23/05/15 Суб 19:41:28 #427 №181856 
>>181461
будет видно
но ты бы прикрепил образец чтоли, а то я не те фильмы смотрю видимо
Аноним 23/05/15 Суб 19:47:10 #428 №181860 
>>181673
ооо боже, я все поня....
Аноним 23/05/15 Суб 20:35:28 #429 №181864 
>>181467

>>181467
>>200 трлн долл. США
>Я прекрасно осознаю сложность возведения целого промышленного космоса на Луне,
ндя? с этого места по подробней, на каких стройках на луне работал. Слона знаешь? держит там притон рядом с большим радиатором.
Аноним 23/05/15 Суб 21:30:17 #430 №181876 
>>181864
Пиздец, что за хуйню ты несешь, в чем твоя претензия? Или может пруф на 200 трлн доставишь?
Аноним 23/05/15 Суб 22:17:36 #431 №181883 
14324086564220.jpg
>>181876
сорри был с телефона, поэтому пришлось образно, так сказать.
я вообще без понятия какие они там планы строят и во что их оценивают,
но подозреваю что в итоге там будет будка в которой будет дежурить 2 космача, и подкручивать гайки и причесывать провода. нах оно нужно таким вот преодолением.

что до претензии, то это не столько претензия
>Я прекрасно осознаю сложность возведения целого промышленного космоса на Луне,
а недоумение, откуда, откуда это прекрасное осознание происходит. зацепило меня это короче, такто понятно вкусовщина, и то что сейчас всякие суммы это вилами на воде. вложат 100 миллиардов, вложат и еще триста если потребуется. эта как продажа бумажки в 1 бакс за 200.

так если на пик посмотреть, то и вообще фигня ценник.
Аноним 23/05/15 Суб 23:29:53 #432 №181902 
>>181876
заметил что там сумма на три порядка больше, но макаба не дала записать
вообщем я бы тоже не отказался от линки на источники, и что они там за эти деньги хотят получить
Аноним 24/05/15 Вск 04:00:35 #433 №181965 
>>179527
Упадёт ли со временем земля на солнце?
Аноним 24/05/15 Вск 04:12:03 #434 №181968 
>>181965
Нет. Солнце раньше упадет на Землю. а точнее пизданёт
Аноним 24/05/15 Вск 11:19:20 #435 №181986 DELETED
Можно ли сделать сверхмощный лазор и взорвать Солнце?
Аноним 24/05/15 Вск 12:59:31 #436 №181995 
>>181965
Земля могла бы упасть на Солнце, в следствии потери энергии через гравитационное излучение, через 10^20 лет, только Солнце до этого превратится в красного гиганта и расширяющейся оболочкой поглотит Землю и расплавит ее в говно.
Аноним 24/05/15 Вск 16:11:08 #437 №182021 
Спейсаны, я правильно понимаю, что гравитационные волны пока не нашли исключительно из-за их слабости? Тут недавно проскакивала какая-то новость об обнаружении гравитационных волн. не?
Аноним 24/05/15 Вск 16:16:21 #438 №182022 
>>182021
понять прост оне могут, что ето. волна, луч, частица, магнитное сочленение частиц или мысли. а если понять не могут, то и обнаружить тоже.
Аноним 24/05/15 Вск 16:58:17 #439 №182026 
>>182021
>Тут недавно проскакивала какая-то новость об обнаружении гравитационных волн. не?
Ребята из BICEP2 заявили, что нашли, норебята, работающие с PLANK, окунули их в говно, потому как результаты весьма сомнительные из-за того, что там, куда смотрели BICEP можно насмотреть только говно.
>из-за их слабости?
Да, это основная проблема.
Аноним 24/05/15 Вск 19:09:21 #440 №182048 
14324837613220.png
>>182021
Все просто: если есть поле, которое взаимодействует с материей (а иначе как бы мы его обнаружили?) , значит это взаимодействие кто-то должен переносить, должен через что-то идти обмен энергией между полем и материей.
С электромагнитным мы разобрались - нашли фотоны разных энергий, научились их генерировать, в общем полный фарш, все включено.
С гравитацией - полный глушняк. Поле есть, взаимодействие есть, но переносчика нет. Предположили, по аналогии с электромагнетизмом - частицу по типу фотона, обозвали ее гравитон, но... ее нигде нет! Никому не удалось обнаружить ни следа ее присутствия.
Отсюда пошли мысли, типа а что если ее и нет? Что, если гравитация - свойство самого пространства, тогда и переносчик не нужен - это просто метрика пространства, как время или расстояние. Оно просто существует и все.
Безусловно, такое предположение не выдерживает никакой критики - это признание поражения, это стыдно. Но пока нет ничего лучше - остается придерживаться этого, помня о том, что это наше поражение, и всеми силами ища выход, дабы оправдать себя.

Так что пока смотри на ТО, но помни, что она неверна, и рано или поздно будет найдено что-то более вразумительное.

Например тот факт, что гравитация меняется со скоростью света, порождая гравитационные волны (которые достоверно зафиксированы спец приемниками, и даже генерируются спецпередатчиками - это старые эксперименты 70-х-80-х), однозначно указывает на то, что переносчик быть должен. А его необнаружимость указывает на то, что мы и так знаем - гравитация с материей связана очень слабо, это что-то среднее между фотонами и нейтрино.
Конечность скорости распространения указывает на то, что гравиворонки немного закручены по вращению планеты, и на то, что для связи гравитация подходит куда хуже радиоволн, но в отличии от них - ничем не экранируется.

Короче вопросов пока больше чем ответов. С гравитацией мы еще не ответили даже на самые базовые вопросы. Мы вывели ее формулу, причем только для одного случая, и на практике - она скорее всего окажется неверна в других случаях. Мы научились детектировать гравитацию, мы научились, пусть и крайне неэффективно, ее модулировать. Но мы не понимаем что это, и как оно работает, мы видим лишь ее проявление, но не понимаем механизма.
Аноним 24/05/15 Вск 20:14:24 #441 №182070 
>>182048
>гравитационные волны (которые достоверно зафиксированы спец приемниками, и даже генерируются спецпередатчиками
Очередной петух с охуительными историями ITT.
Аноним 24/05/15 Вск 20:45:02 #442 №182086 
>>182070
Двачую скептика. Какие детекторы блять?
Аноним 24/05/15 Вск 21:04:46 #443 №182090 
>>182086
Хз, читал в Наука и Жизнь тысяча девятсот лохматого года. Какой-то чувак писал, что засек их с помощью тяжелых подземных цилиндров.
Аноним 24/05/15 Вск 23:10:37 #444 №182112 
>>182090
>что засек их с помощью тяжелых подземных цилиндров.
Чувак был бы сейчас всемирно знаменит, да еще и с нобелевкой, а с гравитационных волн сняли бы приставку "гипотетические".
>Какие детекторы блять?
Детекторов, кстати хватает, но длинна плеча какая-то маленькая. Если бы взлетела LISA было бы охуительно.
Аноним 25/05/15 Пнд 00:46:54 #445 №182136 
>>182112
А он и так знаменит. Именно он придумал все эти детекторы, которых сейчас хватает.
Аноним 25/05/15 Пнд 01:27:08 #446 №182162 
>>182136
Тащемта Вебер напиздел, что что-то там надетектил. Никто не получал таких результатов на аналогичных установках.
Аноним 25/05/15 Пнд 01:47:05 #447 №182175 
>>182162
С чего ты взял, что напиздел? Что увидел - то и сказал. Нет подтверждений, сомнительно - только и всего.
Какой-то поехавший вообще что-то кричал, что если гравиволны существуют, вселенная бы не прожила более 50 млн лет - по моему вообще бред, подсчитали, что луна будет через гравитацию терять энергию дольше, чем до тепловой смерти вселенной.
Гравитация слабо связана с материей, но что-то энергию все-равно переносит. Вопрос - что?
Аноним 25/05/15 Пнд 01:49:12 #448 №182176 
>>182175
Твоя мамаша.
Аноним 25/05/15 Пнд 01:52:01 #449 №182178 
>>182175
нейтрино и переносят
Аноним 25/05/15 Пнд 01:57:26 #450 №182182 
>>182175
С того что если человек говорит, что он получил такие и такие данные, на таком-то устройстве, таким вот методом, а несколько других сделали как он и у них нихуя не вышло - значит человек напиздел, либо надо признавать магию, единорогов и эльфов, каждый раз, когда кто-то говорит, что он их видел.
>>182178
>нейтрино и переносят
Пиздабольство уровня /b/
Аноним 25/05/15 Пнд 02:45:00 #451 №182193 
>>182182
>С того что если человек говорит, что он получил такие и такие данные, на таком-то устройстве, таким вот методом, а несколько других сделали как он и у них нихуя не вышло - значит человек напиздел, либо надо признавать магию, единорогов и эльфов, каждый раз, когда кто-то говорит, что он их видел.
Нет, это говорит о твоем черно-белом методе мышления, и ничего более.
Никто не знает что он там засек, но что-то засек. Это может быть все что угодно - шумы, ошибки, редкое космическое явление. В любом из этих случаев данные не подтвердятся, даже если будут достоверны.
Но это не значит, что человек врет.
А тот факт, что человек отлично разбирается в своей области, и все-равно продолжает настаивать на своем мнении, зная что его обьявят шизофреником, неучем, вруном - чего-то, да стоит. Человек поставил всю свою жизнь и всю свою репутацию на кон - это достойно как минимум уважения.
И не забывай, что не смотря на неподтвержденные данные, разработанными им приборами и принципами продолжают пользоваться даже спустя полвека, тратят на это миллиарды бабла.
И тут выскакивает какой-то школьник и безапелляционно обвиняет такого человека во лжи. Причем посмертно. Занавес.
Аноним 25/05/15 Пнд 03:51:31 #452 №182198 
>>182193
>и все-равно продолжает настаивать на своем мнении, зная что его обьявят шизофреником, неучем, вруном - чего-то, да стоит
Ничего не стоит, как бы он там в чем не разбирался, и какие бы регалии не имел, он остается человеком.
>Человек поставил всю свою жизнь и всю свою репутацию на кон - это достойно как минимум уважения.
Если бы он признал свои ошибки, или сказал, что задетектил узкоколейную дорогу за соседним холмом, или что он там надетектил, то это было бы достойно уважения, а то что он продолжал настаивать на своем, не имея никаких доказательств на руках - говорит о том, что он как раз боялся выглядеть идиотом в глазах коллег.
>продолжают пользоваться даже спустя полвека, тратят на это миллиарды бабла
И ничего не находят. А он там вылавливал гравиволны, как рыбку из пруда.
>И тут выскакивает какой-то школьник
Смажь жопу антипригарным, тебе уже школьники мерещатся.
Аноним 25/05/15 Пнд 06:22:58 #453 №182213 
>>182193
>Человек поставил всю свою жизнь и всю свою репутацию на кон - это достойно как минимум уважения.
Что насчет Ацюковского с Катющеком? Уважаешь пацанов?
Аноним 25/05/15 Пнд 06:35:15 #454 №182215 
>>182198
>Если бы он признал свои ошибки, или сказал, что задетектил узкоколейную дорогу за соседним холмом, или что он там надетектил, то это было бы достойно уважения, а то что он продолжал настаивать на своем, не имея никаких доказательств на руках - говорит о том, что он как раз боялся выглядеть идиотом в глазах коллег.
Fail. Опять ты на других натягиваешь свои скудные модели поведения. А что если он не признал ошибки не потому, что чего-то боялся, а просто потому, что был уверен в том, что видел своими собственными глазами? С чего бы ему в таком случае менять мнение под давлением тех, кто не видел того, что видел он?
Я бы тоже не стал менять мнение, миллионы мух таки ошибаются - я видел то, что видел, другого тут не попишешь, то что вы этого не видели - не моя проблема. С чего мне отрекаться от увиденного собственными глазами ради удовлетворения негодующей толпы? Да пусть хоть пересрется там от своего баттхерта, это не изменит уже свершившихся фактов, а значит и мнение менять бессмысленно. Вот как-то так.
Аноним 25/05/15 Пнд 06:49:48 #455 №182216 
>>182215
Вот смотришь ты на часы, они показывают 14:88. Все остальные смотрят на свои часы, у них 10:21 плюс-минус пять минут. Но ты же сам видел, значит сейчас 14 часов 88 минут, а все остальные просто слепые уёбки. Так?
Аноним 25/05/15 Пнд 07:13:38 #456 №182217 
>>182215
>А что если он не признал ошибки не потому, что чего-то боялся, а просто потому, что был уверен в том, что видел своими собственными глазами?
>то что вы этого не видели - не моя проблема.
>Да пусть хоть пересрется там от своего баттхерта
>мнение менять бессмысленно

У меня от этой хуйни Р. Кеннет Уилсон случился.
Аноним 25/05/15 Пнд 10:07:45 #457 №182238 
>>182048
Бгг. Ну и срач я вызвал простым, казалось, вопросом.
Про чувака, задедектившего гравиволны какими-то подземными цилиндрами, но результаты которого не смогли повторить и, что логично, не признали, впервые слышу. Не соизволит ли анон, произвёдший вброс, дать ссылочку?
Лично мне гипотеза гравитонов и квантовой гравитации пока кажется самой разумной. Вся проблема, как уже было сказано выше, в том, что гравитация - крайне слабое взаимодействие и уловить что-то сидя в гравитационном колодце проблематично.
>Так что пока смотри на ТО, но помни, что она неверна, и рано или поздно будет найдено >что-то более вразумительное.
Это в каком месте она неверна? Если говорить о гравитации, разумеется.
Аноним 25/05/15 Пнд 11:07:56 #458 №182251 
>>182238
>Это в каком месте она неверна? Если говорить о гравитации, разумеется.
Не клеится на квантовом уровне, при охуенном предсказательном выхлопе для макрообъектов.
Аноним 25/05/15 Пнд 11:11:12 #459 №182252 
>>182251
Да, с этим спорить не буду. Но я и не говорил, что "давайте-ка мы применим теорию относительности к гравитонам"
Аноним 25/05/15 Пнд 17:32:18 #460 №182337 
Чем на этих видео поливают ракеты при старте?
https://www.youtube.com/watch?v=1_FXVjf46T8
https://www.youtube.com/watch?v=wcHD9AmkxA0
Что это вообще такое?
Аноним 25/05/15 Пнд 18:21:11 #461 №182340 
>>182337
Водой, чтобы пукан остудить.
Аноним 25/05/15 Пнд 18:21:59 #462 №182341 
14325673192590.jpg
>>182337
Это прозвучит странно, но это шумоподавляющая система. ТТРД больших диаметров (такие как шаттловский, например) дают охуительную вибрацию, т.к. твертое топливо горит неравномерно, как его не ровняй. Собственно, в шаттлах вроде как впервые была применена подобная система гашения звуковых волн. Как известно, в первом запуске шаттла чуть не проебали аппарат вместе с экипажем, из-за неправильно рассчитанной вибрации, повредившей бодифлап.

Зачем это в хуитке с ЖРД, которые горят намного ровней и излишней вибрации не дают - ума не приложу. Но факт есть факт, это система виброгашения стартового стола.
Аноним 25/05/15 Пнд 18:26:41 #463 №182343 
Очень тупой вопрос.
Почему мне пейзаж стартовой площадки КосмосХ напоминает Флориду и спейсцентр им.Кеннеди?
Аноним 25/05/15 Пнд 18:44:26 #464 №182353 
>>182343
Принцип подобия. Не с Байконура же ему срисовывать, при всем уважении к дидам.
Аноним 25/05/15 Пнд 18:47:58 #465 №182357 
>>182343
Потому что целевая аудитория игры - США. А для каждого американца космос = мыс Канаверал. Конечно, вся KSP - это калька с американской космической программы, включая Здание Вертикальной Сборки, пуски над океаном, отсутствие паровозика, конические капсулы, любовь к ТТРД, ЖРД открытого цикла (газоотводная трубка хорошо видна на VT-30 и VT-45), NERVA вместо РД-0410 и т.д.
Аноним 25/05/15 Пнд 18:49:19 #466 №182358 
>>182343
>>182353
В глаза порюсь, пора домой идти.
Аноним 25/05/15 Пнд 18:51:18 #467 №182361 
>>182353
>>182357
А он не там же расположен? Не Во Флориде?
Аноним 25/05/15 Пнд 18:54:34 #468 №182365 
>>182361
Этот стенд во Флориде, да на мысе Канаверал.
http://www.nasa.gov/press-release/spacex-targets-may-6-for-pad-abort-test-of-new-crew-spacecraft
Аноним 25/05/15 Пнд 19:17:57 #469 №182371 
>>182357
Втеки обратно в огурцетред, речь шла о стартовом столе SpaceX.
Аноним 25/05/15 Пнд 19:36:11 #470 №182377 
14325717710720.jpg
14325717710741.jpg
14325717711302.jpg
14325717711773.jpg
>>182357
Ну строго говоря, в ранних версиях KSP и Байконур был. Правда почему-то в горах и с вертикальным VAB вместо горизонтального МИКа. И с него нельзя было пускать, это была пасхалка. Алсо, после введения процедурного ландшафта он оказался под землей.

Тема Old KSC была развита в моде Kerbin-Side, где из него сделали охуительный космодром.
Аноним 25/05/15 Пнд 19:37:01 #471 №182379 
>>182377
Пардон, я сам вытек почем зря, промазал.
Аноним 25/05/15 Пнд 19:57:35 #472 №182382 
>>182341
спавсибо за подробный ответ
Аноним 25/05/15 Пнд 21:28:42 #473 №182403 
>>182341
> Зачем это в хуитке с ЖРД, которые горят намного ровней и излишней вибрации не дают - ума не приложу. Но факт есть факт, это система виброгашения стартового стола.
Дело не в устройстве РД, а в наличие газовой сверхвуковой струи. При начале работы РД (и при последующем истечении) продукты сгорания ударяются о воздух. Так как воздух - среда упругая, то создаются зоны сжатия и разряжения, сиречь звуковые волны. А такое акустическое воздействие не только нехилым образом хуярит по стартовому столу и всему что на нем находится, но и, отразившись, уже ебашит по самой ракете или шатлу. А там много нежного. Поэтому в газовую струю всегда вводят струю воды, чтобы мелкие водяные частички отбирали на себя энергию колебаний воздушной среды.
Аноним 25/05/15 Пнд 23:41:08 #474 №182439 
>>181965
Нет она будет только дорожать.

Где подробно почитать про типы звёзд? Есть ли звёзды, выходящие за пределы главной последовательности?
Аноним 25/05/15 Пнд 23:58:56 #475 №182446 
>>182439
>Есть ли звёзды, выходящие за пределы главной последовательности?
А как же, весь остаток диаграммы Герцшпрунга-Рассела и всякая неведомая ёбаная пиздецома типа объектов Торна-Житков.
Аноним 26/05/15 Втр 00:02:22 #476 №182447 
>>182439
Существование звезд ограничивает не главная последовательность, а физика и их параметры: масса, состав.
Если в звезде не осталось материалов легче железа - она не будет звездой при любом раскладе, т.к. синтез не пойдет. Она может быть, в зависимости от массы, газовым гигантом, пульсаром, нейтронной звездой - куском сверхплотной материи, или черной дырой.
Если легкие элементы есть - ее существование определяется ее массой: она должна быть достаточна для запуска реакции синтеза, если же нет - это будет газовый гигант.
Если масса слишком велика - даже с учетом легких элементов это не будет звездой.
Аноним 26/05/15 Втр 10:46:03 #477 №182498 
Почему у Союза баки под углом, а у Протона - параллельны ракете?
Аноним 26/05/15 Втр 11:44:27 #478 №182504 
>>182498
Обусловлено дизайном ракеты, очевидно боковушки сделали такими, что бы ракета была более аэродинамичной, а баки расположили так, что бы они максимально выгодно заполнили объем.
Аноним 26/05/15 Втр 19:25:26 #479 №182577 
>>182498
Потому что при отделении первой ступени у Протона отваливается вся жопа, а у Союза - только боковушки. Угол нужен, чтобы после сброса нижнего силового пояса за счет продолжающейся работы двигателей создать момент относительно верхней точки крепления боковушек к центральному корпусу. За счет этого момента боковушки разводят в стороны. Гляди видяху.
https://youtu.be/AVvgpKt5uCA?t=7m9s
Аноним 26/05/15 Втр 19:31:01 #480 №182580 
>>182577
Опасно, сложно, у протона проще и безопаснее.
Аноним 26/05/15 Втр 19:34:25 #481 №182581 
>>182580
Да чего там опасного? Голая физика. Всё работает как часы. Не знаю случаев ошибок на это этапе. Хотя, может анон чего слыхал.
Аноним 26/05/15 Втр 19:41:42 #482 №182583 
>>182577
Но ведь есть ракеты, у которых тоже сначала отваливаются боковушки и у них баки расположены нормально, не?
Аноним 26/05/15 Втр 19:51:27 #483 №182585 
14326590879740.jpg
>>182583
Какие например? Бустеры Шатла? Подробностей не знаю, но думаю они просто отстреливаются в стороны. Хотя, повторюсь, тут я не в курсе. У Союза же первое движение боковушек - жопы в сторону, нос прижат. Нужно такое, чтобы вытащить их из замков на центральном блоке. Как только нос вышел из замка - включается РДТТ на кончике боковушки - и отводит нос, обеспечивая вращение уже в другую сторону.
Вот тут в реале можно посмотреть как это происходит.
https://youtu.be/vHWDNrrfhnI?t=2m13s
Аноним 26/05/15 Втр 19:55:53 #484 №182588 
>>182581
Проще - узел разделения только один, стандартный, использующийся давно и везде. А не 8-мь точек разделения и 4 движка, которые должны быть идеально синхронизированы друг с другом, больше узлов - сложнее, больше точек отказа, больше вероятность отказа.
Безопаснее - то, что задница падает вниз, и столкнуться с ракетой не может. А боковуши, при какой-либо проблеме в разделении (позднее выключение, раннее расцепление, и т.п. ошибки тайминга, программные или физические - болт заел например), могут прошить ракету, от которой они отделились. Похожее произошло с шаттолом - его прошил свой же ускоритель, рано отделившийся из-за поломки крепления. С жопой такого не произойдет при всем желании - пока движок работает, она толкает только вверх, топливо выгорело - толкать уже не чем, вторую ступень жопа не догонит при всем желании.
Аноним 26/05/15 Втр 20:00:05 #485 №182589 
>>182588
Ну, с первого взгляда всё верно. Но, дума, были какие-то еще причины применить такую схему в своё время. Может, горячее отделение еще было не до конца отработано или еще чего. Подробностей создания Союза я не знаю, к сожалению.
Аноним 26/05/15 Втр 20:00:45 #486 №182590 
14326596455750.jpg
>>182585
>Какие например?
Американские, китайские, индийские, да много их.
>но думаю они просто отстреливаются в стороны
Почему Союз не отстреливают в стороны, каким образом косая установка, пустых к моменту разделения баков, способствует к исполнению этих пируэтов?
Аноним 26/05/15 Втр 20:05:10 #487 №182591 
>>182590
> Почему Союз не отстреливают в стороны
Повторюсь, наверняка были какие-то технологические ограничения в 50х, которые не позволили просто отстреливаться в стороны. Может система управления была не фонтан, может еще чего. Причин может быть масса.
Аноним 26/05/15 Втр 23:23:49 #488 №182604 
>>182590
>пик с ракетами
Какие-же они огромные. По ящику масштаб вообще не виден, я даже долго думал, что протон и союз примерно одного размера. Я хотел бы, чтобы в моём городе сделали парк, где среди деревьев и тропинок стояли бы макеты самых известных ракет и йоб в натуральную величину.
>Смотри, вот этим нацисты пиздячили Лондон
Уууу, какая здоровая
>А вот на этом летал Гагарин
Ох ебать, вот это дура
>А вот это гептиловое чудовище - рабочая лошадка роскосмова
Ничосе башня!
>А вот это спейс шатол
Да его обходить устанешь!
>А вот эта ебанина пизданула четыре раза на старте
Она больше бабкиной деревни!
>А на этом люди летали на луну
Некоторые дома меньше этих двигателей!
Аноним 26/05/15 Втр 23:58:29 #489 №182609 
>>182604
Было б охуенно кстати. А ещё сделать краники, чтоб из союза пиво наливалось, из протона водяра, из шаттла кола(на запивон), из Сатурна вискарь, ну и так далее.
Аноним 27/05/15 Срд 00:17:01 #490 №182613 
>>182609
Надо чтобы из союза керосин, из шатола жидкий водород с кусочками твёрдого топлива, из протона несимметричный диметилгидразин, аз фау-2, для знающих, так то, что надо.
Аноним 27/05/15 Срд 00:30:53 #491 №182614 
>>182613
Че? Наркоман? Током ебнуть?
Аноним 27/05/15 Срд 15:14:29 #492 №182676 
Всегда ли последнюю ступень сводят с орбиты? Или похуй, пусть летает?
Аноним 27/05/15 Срд 18:04:59 #493 №182704 
>>182676
На низких орбитах - всегда, иначе при неконтролируемом сходе она может ебнуть куда угодно.
На высоких - либо сводят либо отводят на орбиту захоронения, смотря сколько топлива. На орбите захоронения ступень может существовать тысячелетиями.
Аноним 27/05/15 Срд 19:35:13 #494 №182719 
>>182704
Да, на низких сводят. В прошлый раз ракета на поселок ебнулась - полпоселка сожгла к хуям. А если на город упадет?
Аноним 27/05/15 Срд 19:59:54 #495 №182723 
>>182719
эта линк на пол поселка, очень интересно
Аноним 27/05/15 Срд 20:18:03 #496 №182728 
>>182723
Он вероятно эту платину имеет в виду. Только там прямо со старта за бугор пошла, а не с низкой орбиты.
http://en.wikipedia.org/wiki/Intelsat_708
http://www.youtube.com/watch?v=FBJ9ue6GKek
Аноним 27/05/15 Срд 20:24:32 #497 №182732 
>>182728
Лол, что за вопли в начале? Китайский мат?
Аноним 27/05/15 Срд 20:42:05 #498 №182737 
>>182728
Что-то слабо верится, что это всё от одной ракеты. Хотя вспышка не хилая была
Аноним 27/05/15 Срд 20:43:13 #499 №182738 
>>182732
Нет блядь, латышские стихи.
Аноним 27/05/15 Срд 21:13:14 #500 №182740 
>>182737
1000 тонн отборной высокооктановой топливно-окислительной смеси - это пиздец само по себе, достаточно одной искры... Даже просто хуйнуть 1000 тонн песка или воды по населенному пункту - все дома посносит, безо всякого огня и взрывов.
А если такую шнягу разогнать и хуйнуть об землю - получится целый огненный цунами, огромная волна огня, с температурой в 1,5-2 килоградуса, быстро движущаяся от точки удара по вектору движения ракеты, сжигающая все на своем пути - она может пройти сотни метров, прежде чем догорит. Спасения нет нигде - даже если спасешься от огня где-нибудь в подвале, пожгет температурой - вокруг весь воздух нагрет до тех самых 1,5-2-х килоградусов. Плюс остаточное горение. Потребуется что-то, что сможет выдержать такую температуру в течении пары часов, не пропустив ее внутрь, при этом не имеет обмен воздухом с окружающей средой - какой-то герметичный бункер с толстыми стенами и запасом воздуха. У жителей просто не было шансов.
Даже просто подорванный бак от автомобиля забрызгивает все вокруг в радиусе 50-100 метров горящим топливом, а тут и топлива намного больше, и импульс большой, да еще все обильно сбрызнуто жидким окислителем - это пострашнее огнемета, т.к. горит интенсивнее, струя гораздо горячее, да и просто топлива дохуя.
Аноним 27/05/15 Срд 21:20:54 #501 №182741 
>>182740
>высокооктановой топливно-окислительной смеси
Там не керосин, Великий Поход-3B гептиловый в основном, по типу Протона, только полегче. Что впрочем дела не меняет. Это их аналог нашей Неделинской катастрофы с Р-16.
Аноним 27/05/15 Срд 21:22:48 #502 №182742 
>>182737
>Что-то слабо верится, что это всё от одной ракеты.
Йоба-то нехилая, 425 тонн масса на старте + объемный взрыв. То что не прореагировало, потравило гептилом.
Аноним 27/05/15 Срд 22:45:14 #503 №182756 
14327559141810.jpg
Есть уже карта новых участков вселенной? Свет уже явно дошёл.
Аноним 27/05/15 Срд 22:55:25 #504 №182760 
14327565254470.jpg
>>182756
Аноним 27/05/15 Срд 23:01:10 #505 №182762 
>>182760
Да думаю может с прошлого поста умные люди появились, видимо нет.
Аноним 27/05/15 Срд 23:24:02 #506 №182769 
>>182762
Ты зато дохуя умный? Ну-ну.
Аноним 28/05/15 Чтв 07:54:34 #507 №182802 
14327888743150.jpg
>>176350
Так, посони, важный вопрос.

Есть одно зеркало, нужно его в космос закинуть.
Это зеркало от БТА, 6 метров, например.

Чем это сделать?
Аноним 28/05/15 Чтв 08:01:27 #508 №182803 
>>182802
Вдогон: желательно отечественный носитель, можно эксгумировать.
Аноним 28/05/15 Чтв 09:12:40 #509 №182810 
>>182756
Новых участков НЕ БУДЕТ. Вселенная разбегается.
Аноним 28/05/15 Чтв 09:33:34 #510 №182811 
>>182810
Вранье
Аноним 28/05/15 Чтв 10:36:48 #511 №182818 
>>178216
Пингвины ссут тебе в ротешник
Аноним 28/05/15 Чтв 10:38:05 #512 №182819 
>>178217
Ой промазал. Это

>>182818
Этому.
Аноним 28/05/15 Чтв 10:45:10 #513 №182821 
>>179478
Тоже спрошу про это кино.

Когда МакКонахи намеренно падает в ЧД и удаляется от Индюренс почему баба еготслышит. Ведь чем ближе к ЧД, то тем больше её влияние на время. То есть время для них должно идти по разному. Но она ведь слышит его в прямом эфире до тех пор пока он не пересек горизонт событий. Где подвох?
Ну и ещё вопрос: ОТКУДА БЛДЬ В ЧД ЕБАНЫЕ КНИЖНЫЕ ШКАФЫ? Нахуя надо было так поротб конец фильма?
Аноним 28/05/15 Чтв 10:46:26 #514 №182822 
>>179754
Вопрос на засыпку. Если я докажу наличия другой вселенной то мне дадут бабосы и нобелевку?
Аноним 28/05/15 Чтв 10:47:14 #515 №182823 
>>181727
Нельзя просто "зависнуть" на 200км. Ты в любом случае будешь летать вокруг планеты с охуевшими скоростями. Чтобы тебе начать падать вертикально вниз с такой высоты, то надо будет моментально погасить горизонтальную скорость.
Аноним 28/05/15 Чтв 11:53:41 #516 №182832 
>>182802
Нахуя, проще сделать новое современное зеркало из более легкого материала. Вон у Джеймса Уэбба 6 метров зеркало, а масса 6,2 тонны, со всей хуйней, в отличии от БТА с зеркалом в 40 тонн.
>>182803
>можно эксгумировать.
Тогда Энергией выводить, другие не поднимут.
Аноним 28/05/15 Чтв 12:02:03 #517 №182834 
>>182822
Да, такое открытие - 100% нобелевка, но ты не докажешь.
Аноним 28/05/15 Чтв 12:08:05 #518 №182835 
>>182834
Абсолютно ты утверждать этого не можешь, что не докажу.
Аноним 28/05/15 Чтв 12:14:25 #519 №182836 
В космосе все перемещается по эллиптическим орбитам и.е. что-то вращается вокргу чего-то. Чем больше скорость перемещения объекта (ну, допустим, зонда с бесконечным топливом) тем его траектория становится все более похожей на прямую, так? Т.е. теоретически, можно слетать на марс по прямой безо всяких гомановских переходов и прочих эллиптических орбит, просто это оверзатратно по топливу? Так вот, при какой скорости траектория движения объекта становится абсолютно прямой? ведь если бы мы увеличили масштаб для того зонда, то получилось бы, что его траектория Земля-Марс прямая, а если взгянуть подальше, и если бы он пролетат мимо Марса, то его траектория "Наша солнечная система - соседняя солнечная система" была бы уже далеко не прямой.. Неужели на все ответ - скорость света?
Аноним 28/05/15 Чтв 12:40:57 #520 №182842 
>>182821
>Но она ведь слышит его в прямом эфире
Там отсутствует одновременность этих событий.
http://www.youtube.com/watch?v=3qYx44JCDmA&index=38&list=PLk0Hd0vmvFM-cKdiEGoMso7L7leei0TH4
Все, что она слышит - какие-то обрывки, которые догоняют ее.
>ОТКУДА БЛДЬ В ЧД ЕБАНЫЕ КНИЖНЫЕ ШКАФЫ?
В фильме прямым текстом объясняется что это за "шкафы" и откуда они взялись.
Аноним 28/05/15 Чтв 12:47:16 #521 №182844 
>>182835
>Абсолютно ты утверждать этого не можешь, что не докажу.
Вперед, че. Можно собрать кучу свидетельств в пользу этой гипотезы, но доказать существование соседнего пузыря, не выбравшись из своего - невозможно.
Аноним 28/05/15 Чтв 12:54:49 #522 №182849 
>>182844
Принципе ты прав. А за полкрепленную математическим доказательством теорию бабло дадут?
Аноним 28/05/15 Чтв 13:11:03 #523 №182855 
>>182849
>А за полкрепленную математическим доказательством теорию бабло дадут?
Математически подкреплено, что ЧД существуют, но за ЧД никто нобелевку не получил, до сих пор гипотетический объект.
Аноним 28/05/15 Чтв 16:53:26 #524 №182884 
>>182836
Бамп епта
Аноним 28/05/15 Чтв 17:29:10 #525 №182889 
>>182884
Да, все верно.
ПЕРЕКОТ Аноним 28/05/15 Чтв 17:35:42 #526 №182892 
Перекат, умники.

https://2ch.hk/spc/res/182891.html
https://2ch.pm/spc/res/182891.html
>>182891
Аноним 28/05/15 Чтв 18:19:52 #527 №182907 
>>182842
Откуда
Аноним 28/05/15 Чтв 19:11:20 #528 №182924 
>>182836
> при какой скорости траектория движения объекта становится абсолютно прямой?
При беконечной. Сравни: идешь ты такой по окружности. При каком радиусе этой окружности ты пойдешь прямо?
Аноним 28/05/15 Чтв 21:25:44 #529 №182990 
>>182924
Получается, что при скорости света, коли он всегда движется по прямой про гравитационное линзирование умолчим
Аноним 29/05/15 Птн 00:34:36 #530 №183026 
>>182822
нет, так как никто не поверит пока ты динозавров от туда не начнешь вытаскивать, по одному динозавру на десять скептиков. при таких раскладах тебе будет лучше самому эксплуатировать ее. но если тебе надо будет для ее существования чтобы все чтото сделали - там скрестили пальцы на ногах в час Пи, то в этом случае печаль, можно считать что ее не существует.
Аноним 29/05/15 Птн 12:35:09 #531 №183109 
Спутник ГСО неподвижен относительно поверхности земли, но крутится относительно звезд. А если тело беудет неподвижно относительно звезд, что тогда? Упадет?
Аноним 30/05/15 Суб 11:00:50 #532 №183327 
>>183109
Он не неподвижен. Он летает вокруг земли. Просто его скорость равна скорости вращения земли вокруг оси.
comments powered by Disqus

Отзывы и предложения